1. The course called Outdoor Emergency Care was created by the National Ski Patrol to:
• Provide a standard of training for OEC technicians and others involved in outdoor recreation.
2. The person who first organized a volunteer ski patrol and then was asked to organize a national patrol was:
• Minnie Dole.
3. The 10th Mountain Division was founded:
• After Minnie Dole convinced the U.S. Army that a winter warfare unit would be valuable.
4. The first emergency responder textbook created exclusively for ski patrollers was titled:
• Winter Emergency Care.
5. Dr. Warren Bowman is considered the father of Outdoor Emergency Care because he:
• Created the concept of intermediate medical care for ski patrollers.
6. The 6th Edition of Outdoor Emergency Care:
• Contains the knowledge and skills identified by the NHSTA for Emergency Medical Responder training plus additional information and training for patrollers.
7. In order to maintain OEC certification, you must:
• Complete the assigned refresher course each year that covers one-third of the curriculum every year during the three year refresher cycle.
8. Good Samaritan laws generally:
• May apply to people who do not receive compensation for helping people.
9. Although regulations may differ from state to state, the general laws that may protect OEC technicians from liability while providing emergency care to a patient are known as:
• Good Samaritan laws.
10. You have just finished your shift, and while driving home still wearing your patrol jacket, you come upon a car collision in which people appear to be injured. You recognize that if you approach the collision wearing your patrol jacket, members of the public may have a reasonable expectation that you will provide care. This expectation is known as:
• The doctrine of public reliance.
11. While you are alone and caring for an injured skier in the aid room, a fellow patroller calls and tells you that your own young child was just injured in the terrain park. Concerned for your child, you leave the patients care unfinished and run to the terrain park. Your action could be viewed as:
• Abandonment.
12. The harming of an individual by not performing up to the technicians standard of training is called:
• Negligence.
13. A situation in which an OEC technician leaves a patient before the patients care is transferred to another qualified individual is considered:
• Abandonment.
14. You are skiing down your favorite slope and notice a small group gathered around a middle-aged man sitting on the ground. You note that the boarder is holding his wrist and appears in pain. He agrees to have you examine him but then refuses any more help. You explain the possible consequences of not splinting the wrist and of the boarder trying to get down the mountain without help. This explanation of the risk of refusing care is provided in order to:
• Diminish your risk of being accused of abandonment.
15. Which of the following statements concerning standard of training is true?
• Acting within the standard of training is fundamental to reducing an OEC technicians legal risk.
16. Which of the following is not considered a form of patient consent?
• Absolute consent
17. Upon finding an unconscious patient at the scene of a bad accident, your next step should be to:
• Assume implied consent and begin to examine and treat the patient.
18. The type of consent that is based on an appreciation and understanding of the facts, implications, and possible future consequences of an action is known as:
• Informed consent.
19. Which of the following descriptions is the best example of expressed consent?
• A 52-year-old skier who comes to the aid room and asks for help
20. What type of consent would apply for an unresponsive 28-year-old male victim of a motorcycle collision?
• Implied consent
21. You arrive at the scene of an accident where a 22-year-old female skier has hit a tree and has a large bleeding cut on her head. When you ask her if she needs help, she refuses care and says she just needs to sit for a while. Noticing the amount of bleeding, you calmly express your concern for her, but she continues to say she doesnt want help. You hesitate to touch her because you are concerned that if you do, she could accuse you of:
• Battery.
22. A network of specially trained personnel, equipment, facilities, and other resources that respond to medical emergencies regardless of cause, location, or the patients ability to pay best describes:
• An emergency care system.
23. Which of the following EMS attributes is most effective for reducing the incidence of injury or illness in skiing or other outdoor activities?
• Prevention
24. Which of the following attributes is not one of the 14 attributes of an effective emergency care system?
• Public acceptance
25. Which of the following is not an attribute of an emergency care system?
• Equipment
26. Which of the following levels of emergency personnel emphasizes the provision of immediate life-saving treatment and stabilization to critically ill or injured patients while waiting for additional emergency response?
• Emergency medical responder
27. An OEC technician meets or exceeds the NHTSA national training requirements of:
• An emergency medical responder.
28. Most emergency care systems deploy personnel to emergencies using a stratified approach based on the patients condition or anticipated needs. Which of the following lists presents the normal order of deployment of responders?
• EMR, EMT, AEMT, paramedic
29. Which of the following is not one of the four nationally recognized prehospital emergency care provider levels?
• Outdoor emergency care technician
30. Many emergency care systems deploy personnel to emergencies using a tiered approach. OEC technicians are most likely deployed in which tier?
• Tier 1
31. Critical care providers who bring advanced life support equipment, therapies, and interventions to the scene are typically deployed in what tier?
• Tier 4
32. The seamless delivery of high-quality emergency medical care as patient transitions from initial contact with an EMT through definitive treatment best describes:
• Continuity of care.
33. Which of the following choices is not an example of direct medical oversight?
• Written protocols
34. Which of the following characteristics is not a characteristic of indirect medical control?
• Communication by radio
35. Guidelines developed at the local level by emergency personnel familiar with an areas medical needs, available resources, system capabilities, and local standard of care are:
• Protocols.
36. Written instructions that indicate what should be done in a given situation are best described as:
• Protocols.
37. The reason it is important that OEC technicians participate in quality improvement programs in services in which they work or volunteer is:
• To identify changes that, if needed, will ensure the future care a patient receives in is accordance with local protocols and guidelines.
38. Which of the following methods is the best method for ensuring that the treatment rendered by OEC technicians meets or exceeds customer expectations and national education standards for emergency medical personnel?
• Quality improvement
39. Sometimes the most important thing in a rescue situation is to:
• Take care of yourself.
40. Mental preparedness comes best if you:
• Have a positive attitude to help deal with difficulties.
41. During a very strenuous rescue, you may need to eat:
• Hourly.
42. The best direction of moisture transfer in the base layer is:
• Outward.
43. The best strategy to maintain warmth in the cold winter months is to wear:
• Clothing in three layers; a base to retain heat, a middle insulating layer, and a water and wind-repelling outer layer.
44. Which of the following SPF ratings would provide the greatest level of protection?
• 50
45. Some vector-borne illnesses are transmitted to humans via:
• Deer ticks.
46. Wearing clean gloves, an impervious gown, and goggles while cleaning a long-board contaminated with blood helps protect the OEC technician from which form of potential infectious disease transmission?
• Indirect contact with an object contaminated with pathogens
47. Pathogens from a scene can harm rescuers if they:
• Invade the body.
48. The practice of protecting yourself from exposure to body fluids based on the assumption that all patients are potentially infectious is referred to as:
• Standard precautions.
49. Which one of the following techniques is one of the most effective techniques that OEC technicians can use to prevent the spread of infection?
• Washing their hands
50. When caring for a patient who was stabbed with a knife in the ski areas bar (a crime scene situation), it is important for the OEC technician to do all of the following to support chain of custody except:
• Wrap the knife found on the floor next to the patient in plastic and hand it to the EMTs so they can show the length of the knife to the Emergency Department staff.
51. Which one of the following statements about the Incident Command System is true?
• ICS operates with standardized terminology to avoid confusion.
52. Of the five functional areas within the Incident Command System (ICS) structure, the first to be established should be:
• Incident command.
53. The primary function of the incident commander is to:
• Provide overall leadership and direction.
54. The incident commander has assigned you to be the operations section chief. In this role, you are responsible for all of the following except:
• Determining incident objectives and strategy.
55. Which of the following tasks is not a responsibility of the incident commander?
• Triaging the injured patients
56. All of the following are functional areas of the incident command system structure except:
• Transportation section.
57. In the incident command system, the operations section is responsible for:
• Executing the strategy of the incident action plan.
58. A formal, organized method for managing an incident, regardless of its cause, size, scope, or complexity, is called:
• An incident command system.
59. In the incident command system, which of the following sections is responsible for maintaining all incident reports except for financials?
• Planning section
60. The person responsible for maintaining equipment and ensuring that facilities meet specified needs is the:
• Logistics chief.
61. In the incident command system, the section responsible for maintaining injury, death, and damage documentation as well as maintaining reimbursement records is the:
• Finance/administration section.
62. Which one of the following triage tag indicates the lowest priority for care, expectant, for a patient at a multiple-casualty incident using the ID-ME triage categories?
• A black/gray tag
63. You are transporting a patient at a multiple-casualty incident. The patient has a yellow triage tag. This tag means:
• Treatment of the patient could be delayed.
64. The most widely used triage categorization system in the world is ID-ME. This acronym represents the four specific triage categories used by NATO forces, National Disaster Life Support, public safety agencies, and search and rescue groups. The ID-ME triage categories are:
• Immediate, delayed, minimal, and expectant.
65. At a multiple-casualty incident that is using the START Triage method, you are brought a patient with a red tag tied to his wrist. You should recognize which one of the following?
• Immediate care and transport of the patient are necessary.
66. You are at an outside skiing exhibition when a spectator bleacher collapses. Several injured people are walking around with various injuries. Which of the following instructions is most appropriate for these ambulatory patients?
• Go and wait by the ticket booth at the bottom of the slope.
67. There is a fight involving approximately 20 bar patrons. Weapons were used, and there are varying degrees and types of injuries. The first person on scene has started assessing a person lying on the floor with blood covering his shirt. Quick assessment reveals him to be breathing at 24 times per minute. Which of the following actions should be done next using the START triage method?
• Check for a radial pulse or capillary refill.
68. At the scene of a multiple-casualty incident, you are presented with a patient who is not breathing. According to the Simple Triage and Rapid Transport (START) system, which of the following actions should you take next?
• Open the airway.
69. You are triaging using the Simple Triage and Rapid Transport (START) system, and you find a patient who is breathing 18 times per minute and has a radial pulse. Which one of the following actions should you take next?
• Check to see if the patient can follow simple commands.
70. In using the Simple Triage and Rapid Transport (START) system of triage, which one of the following patients should have a yellow tag applied before being moved to the treatment area?
• A female with a respiratory rate of 22 and a palpable radial pulse who squeezes your fingers when instructed to do so
71. A teen involved in a multiple-casualty incident is found lying on the ground with an obvious deformity to his left thigh. Using the START system for triage, you note that he is not breathing. Which one of the following should you do next?
• Open the patients airway; next check for a radial pulse.
72. While in charge of triage at a scene involving 30 patients exposed to carbon monoxide, you find a young female who is not breathing. After you open her airway and find that breathing does not return, you should:
• Place a black tag on her and move on to the next patient.
73. When you are placed in charge of the treatment unit at the scene of a multiple-casualty incident, which of the following four patients will you treat first?
• A confused 69-year-old male with a respiratory rate of 40 and a rapid and weak radial pulse
74. In the acronym START, the letter S stands for:
• Simple.
75. When properly trained in the START system, an OEC technician should be able to complete a patient assessment in less than:
• 30 seconds.
76. To lessen ones chances of injury while lifting and moving a patient, one should:
• Combine good lifting and moving techniques.
77. To maintain proper alignment when moving or lifting a heavy object, the weight of the object must be:
• Evenly transferred to the legs.
78. When a person exhibits good body mechanics and lifts properly, the spine is:
• Straight and in an aligned position.
79. Extremity lifts may not be tolerated by:
• Elderly patients and patients with respiratory disorders.
80. Which of the following instructions is most important for the use of good body mechanics in a power lift?
• Straighten your legs to lift.
81. Like a direct ground lift, a ________ lift is used to raise patients who are lying on their back.
• BEAN
82. With a BEAN lift, the patient is:
• Raised for transfer onto a backboard.
83. A bridge lift is appropriate when a patient:
• Has no suspected spinal injury.
84. An urgent move is required when:
• The rescuer and the patient must move to a safer location.
85. To use an urgent move called the shoulder drag, you should:
• Grab the patients clothing at the shoulders, support the head, and then bend your knees and drag the patient.
86. One of the most common nonurgent moves for OEC technicians to use when alone is the:
• Human crutch.
87. A basket stretcher is a good method for transporting a patient:
• Out of the backcountry.
88. The first step in placing a patient in the toboggan for transport is:
• Deciding whether or not to put the patients injury uphill.
89. The uphill positioning principle has exceptions. For patients with breathing difficulty, the patient should be placed either seated or uphill depending on other injuries. The uphill concept allows easier breathing because:
• Of less pressure from the abdominal contents pushing upwards on the thorax.
90. A woman who is six months pregnant and experiencing back pain should be placed on a backboard and then placed in a toboggan in which of the following positions?
• On her left side
91. An appropriate landing zone (LZ) for safely landing a helicopter is an open area that is approximately:
• 100 feet x 100 feet.
92. During helicopter landing or takeoff, most protocols only allow the _______ to be in the LZ.
• Landing coordinator
93. After a landing helicopter is safely on the ground, one may approach the aircraft only when signaled by the:
• Pilot or flight crew chief.
94. When approaching most helicopters, whether running or not, the safest way is to do so from the:
• Front.
95. Which of the following instructions for approaching a running rescue helicopter is most dangerous?
• Remain in an upright position and approach from the rear.
96. The study of human and animal structures at the gross and microscopic levels is known as:
• Anatomy.
97. The study of mechanical, physical, and biochemical functions of humans is known as:
• Physiology.
98. Which one of the following anatomical statements is true?
• The knee is proximal to the ankle.
99. After examining his patient, your partner tells you that he believes the patient has hyperflexed the knee. What has happened if a patient has hyperflexed a knee?
• The patient has bent the knee backward beyond its normal range of movement.
100. You arrive at an accident scene and find a patient lying on the ground with her right arm straight out to the side (at a 90-degree angle to her body). You ask her if she is able to pull the arm toward her body, but she cannot. You would document this as an inability to __________ her right arm.
• Adduct
101. The term used to describe the action of taking an extremity away from the midline of the body is:
• Abduction.
102. The wrist is:
• Distal to the elbow.
103. You arrive at the scene where a patient has a possible fracture of the left hip. You note that she is lying on her back and her left leg is turned outward. You would document this position as:
• Supine with external rotation of the left lower extremity.
104. You apply and use an AED on a patient lying on his back. His pulse returns, and he is breathing adequately. What position is he in?
• Supine
105. Which one of the following descriptions best describes the normal anatomic position?
• Standing with the arms down at the sides and the palms facing forward
106. A man lying in the prone position is:
• Lying face down.
107. You are assisting in the care of an elderly patient complaining of shortness of breath. You place the patient on 15L of oxygen via a nonrebreather mask and place him in a high Fowlers position. This means that you will place this patient on his:
• Back, with his upper body elevated at 90 degrees from the waist.
108. Blockage of which of the following prevent air flow into and out of the lungs?
• The trachea
109. Which of the following statements best describes the function of the respiratory system?
• Providing the body oxygen
110. The structure containing the vocal cords is the:
• Larynx.
111. The leaf-shaped flap that helps prevent food from entering the lower respiratory system is called:
• The epiglottis.
112. Which one of the following lists represents a correct sequence for the passage of air into the lungs?
• Mouth, pharynx, trachea, bronchi, alveoli
113. Under normal circumstances, carbon dioxide is excreted from the body by what system?
• The respiratory system
114. Which of the following systems is not a body system?
• The thoracic system
115. The diaphragm separates the thoracic cavity from the:
• Abdominal cavity.
116. When the diaphragm and intercostal muscles relax, which one of the following occurs?
• Exhalation
117. Which one of the following events causes an individual to take a breath?
• The diaphragm contracts.
118. The primary function of the heart is:
• Pumping blood throughout the body.
119. Which of the following structures brings oxygen-depleted blood to the right atrium?
• The venae cavae
120. Which chamber of the heart is responsible for pumping blood to the lungs?
• The right ventricle
121. Following chemotherapy, a low white cell count increases a patients risk for which of the following?
• Infection
122. The liquid that blood cells and nutrients are suspended in is called:
• Plasma.
123. The left atrium:
• Receives blood from the pulmonary veins.
124. The nervous system is subdivided into which two main parts?
• The central and peripheral nervous system
125. A patient has suffered damage to his brainstem. As a result of this injury, which of the following signs might you expect to see?
• Abnormal respirations
126. The pressure exerted on the inside walls of arteries when the left ventricle contracts is called:
• Systolic pressure.
127. Cerebrospinal fluid provides three beneficial properties. Which of the following is one of those properties?
• Helps cushion the brain
128. As a patient gives her medical history to you, which of the following would you relate to the endocrine system?
• Removal of the thyroid
129. The thyroid glands, adrenal glands, pituitary glands, and gonads are part of the:
• Endocrine system.
130. A patient has suffered a burn to the skin. Based on the functions of the skin, to which of the following conditions is the patient most susceptible?
• Infection
131. Which of the following statements regarding the integumentary system is true?
• The skin protects the body against bacteria and other pathogens.
132. Which of the following is the largest organ of the body?
• The skin
133. What body system provides support and structure to the body?
• The skeletal system
134. The terms occipital, frontal, and parietal refer to what part of the body?
• The cranium
135. A new patroller tells you that he injured a tendon above his patella three years ago. You recognize that this injury involves a structure that:
• Connects muscles to the patella.
136. Which of the following groups of bones could be involved in a patient with a broken leg?
• Femur, tibia, fibula
137. The lower jaw is also called the:
• Mandible.
138. The bones of the upper extremities include the:
• Humerus and radius.
139. Which of the following lists identifies the regions of the spinal column from superior to inferior?
• Cervical, thoracic, lumbar, sacral, and coccyx
140. Which of the following is a function of the skeletal system?
• Producing blood cells
141. A patient experiencing thoracic spine pain has pain in his:
• Upper back.
142. You are working on a patient who has skied into a tree and are told that there is a frontal hematoma. From that, you expect to find the injury where?
• The skull
143. A patellar fracture affects which part of the body?
• The knee
144. A female patient suffers from a muscular disease and cannot walk. Based on this fact, you should recognize which of the following muscle types is affected?
• Skeletal muscle
145. The pelvic cavity contains which of the following organs?
• The rectum and the reproductive organs
146. The spleen is part of which body system?
• The lymphatic system
147. Which of the following lists presents the order in which an OEC technician should assess a patient in a field setting?
• Scene size-up, primary assessment, secondary assessment
148. Which of the following statements would indicate that an OEC technician is ready to begin the first phase of a patient assessment?
• The scene is safe and appears to be free of hazards.
149. Which of the following things does not indicate a possible safety issue during a scene size-up?
• ABCDs
150. You have been dispatched to an 89-year-old female with an unspecified complaint. When assessing this patient, which one of the following will the OEC technician do as quickly as possible to develop a better understanding of the emergency?
• Determine if the chief complaint is medical (NOI) or trauma related (MOI)
151. You are interviewing a prospective candidate for the position of OEC technician with your service. During the interview, you ask the applicant to describe the purpose of the primary assessment. Which of the following best describes that purpose?
• To identify and treat life-threatening conditions
152. You have been dispatched for an elderly male complaining of shortness of breath. When should you start the process of forming a general impression about this patient?
• While you and your partner are approaching him
153. Forming a general impression is done before which phase of patient assessment?
• Primary assessment
154. You arrive at the scene of a fall, where a 42-year-old woman fell backward off a stepladder while cleaning windows. She is lying on the ground, complaining of pain to her ankle. She tells you, If I had just been more careful and moved the ladder instead of reaching, this never would have happened! Based on this information, which of the following can you conclude?
• She has an open airway, is breathing, and has circulating blood to her brain.
155. A 36-year-old patient on the AVPU scale who has overdosed on an unknown drug is breathing and will not open his eyes, even when his name is loudly called. Which one of the following should the OEC technician do next?
• Perform a shoulder pinch
156. The "P" in "AVPU" stands for what?
• Pain
157. If you have to clap your hands loudly in a patient's face to get some response, what letter is scored on AVPU?
• V
158. Which describes a patient scoring U on AVPU?
• Does not respond to pain
159. You are by the side of a patient who is unresponsive and has a history of heart failure. When assessing the airway, which one of the following observations best indicates an open airway?
• You can hear normal respirations and see the chest rise and fall.
160. How should an OEC technician best determine the adequacy of a patients breathing during a primary assessment?
• Look for the rise and fall of the patients chest.
161. Which of the following statements indicates that an OEC technician correctly understands capillary refill?
• For a capillary refill test to be normal, it must be assessed at room temperature and be less than two seconds.
162. A chief complaint is defined as the:
• Primary reason the person is seeking medical care.
163. A 44-year-old male was on a ladder cutting limbs from a tree when he fell. He is found to be unresponsive with normal respirations. His breathing is adequate at a rate of 12 per minute and his carotid pulse is strong at 88 per minute. Following the primary assessment and initial management, which one of the following actions will the OEC technician complete while performing the secondary assessment?
• Get a medical history from a family member
164. An OEC technician should obtain the medical history of an alert and oriented trauma patient prior to the:
• Secondary assessment.
165. An OEC technician identifies the medications a patient takes during which portion of a patient assessment?
• The medical history
166. Signs and symptoms can be identified using the:
• SAMPLE acronym.
167. Which of the following findings is a sign, not a symptom?
• An open leg fracture
168. Which of the following findings is a symptom?
• Ringing in the ears
169. Which of the following responses is an appropriate response for the letter P in the OPQRST mnemonic for a patient with abdominal pain?
• It hurts worse when I take a deep breath.
170. What letter in a SAMPLE interview indicates that a patient has a history of seizures?
• P
171. What letter in a SAMPLE interview indicates that a patient had pancakes for breakfast two hours ago?
• L
172. As you assess a patient for a medical emergency, the patient states the pain is in their chest up to the shoulder and down the left arm. To what letter of OPQRST would this apply?
• R
173. What letter of the OPQRST mnemonic reflects a medical emergency patients report that difficulty in breathing is worse while attempting to walk?
• P
174. What letter of the OPQRST mnemonic reflects a medical emergency patients report that difficulty in breathing became worse about two hours ago?
• T
175. During assessment of a responsive medical patient, you gather a medical history using the memory aid SAMPLE. To obtain information related to M, which one of the following questions should you ask?
• Are you currently taking any medications?
176. The finding that a patients skin is warm, pink, and dry during a primary assessment suggests:
• Normal circulation.
177. A secondary assessment consists of which of the following steps?
• Hands-on physical exam and vital signs
178. DCAP-BTLS is mnemonic to assist you in remembering:
• What to assess while inspecting and palpating a patient.
179. Which of the following actions should you take during a secondary assessment of a 45-year-old female with a severe headache?
• Assess from head to toe, obtain a SAMPLE history, and obtain vital signs
180. A secondary assessment (including vital signs) should take about how long?
• 2-5 minutes
181. The D in DCAP-BTLS stands for:
• Deformity.
182. An easy way for an OEC technician to remember what to look for in assessing injuries in a trauma patient is the:
• Mnemonic DCAP-BTLS.
183. As you assess a patient using the DCAP-BTLS mnemonic, the letter that denotes edema of the ankle is:
• S.
184. Your patient is 5 years old and has fallen, resulting in a scraped knee. What letter indicates the scraped knee in the DCAP-BTLS mnemonic?
• A
185. You are performing a secondary assessment on the unrestrained adult passenger of a motor vehicle that rolled several times at a high rate of speed. The patient has a skull laceration and is responsive to painful stimuli and in a state of hypoperfusion. When assessing the head, which one of the following is appropriate?
• Checking the pupils with a penlight
186. During assessment, a patient's eyes should move how?
• In unison
187. Which one of the following signs should an OEC technician detect during a secondary assessment of a critically injured patient?
• Bruising and tenderness to the abdomen
188. A complete set of vitals is taken at the:
• Beginning and end of the secondary assessment.
189. You are assessing an eight-month-old whose mother states has been vomiting for two days and not eating or drinking. When assessing the pulse in an infant, which site should you check first?
• The brachial artery
190. Which of the following tasks is part of a secondary assessment?
• Taking a blood pressure reading
191. Identify the two types of medical communications that are most important to inform other medical providers of a patients condition and progress.
• Oral communication and written documentation
192. The OEC technician can verbally report to others in a timely method about an incident using the acronym SAILER. The acronym is best represented by which one of the following?
• The gender and age of the patient, the chief complaint of the patient, your location, the equipment you need, and the other resources you need
193. When a patient arrives at the first-aid station, a status of the patient is given. The name of the brief oral report is called what?
• Handoff report
194. Which of the following documents may be used as a medical-legal record for patient care?
• Field notes of the patroller, patient care report (PCR), incident report forms
195. An incident report form is provided by the areas insurance carrier. What is the purpose of the incident report form?
• To collect data surrounding the patient and incident, gather patient care data, and to communicate medical information to other parties
196. The patient care report (PCR) is best organized and completed by using which of the following methods?
• SOAP and CHEATED
197. Please select the most effective components that the OEC technician uses in medical documentation and that best represent the primary parts of the assessment and management process.
• Chief complaint, history, examination, assessment, treatment, evaluation, and disposition
198. When a patient refuses care, under what part of the CHEATED acronym does that fall?
• D
199. What part of CHEATED includes the physical exam of the patient?
• E
200. On which form would you be most likely to use the SOAP or CHEATED acronyms?
• Patient care report
201. The PCR (patient care report) is a legal medical document completed by the OEC technician. Therefore, good written documentation is important. In the list below, choose the answer that represents the components of a good written case report.
• Appropriate medical terminology
202. The lower airway consists of which of the following structures?
• Trachea, bronchi, and alveoli
203. Which of the following patients has an actual or potential critical occlusion of the upper airway?
• 3-year-old with a fever and swelling of the larynx
204. You want to open your patients mouth to check for obstruction with the cross-finger technique. Which of the following describes the best way to accomplish the cross-finger technique?
• Place your index finger on the upper teeth and thumb on the lower teeth. Then spread your finger and thumb apart opening the mouth.
205. Your patient has just had a seizure and is presenting with snoring respirations. You will open her airway with the head tilt and chin lift method. Which option correctly describes this method?
• Place one hand on the forehead and one hand on the boney part of the lower jaw.
206. You are correctly performing the jaw-thrust maneuver when you:
• Use the thumbs or fingers as a lever to lift the patients mandible forward.
207. You are caring for an unconscious patient who has vomited and appears to have a blocked airway. Using the cross-finger technique, you open the patients mouth and can see a large piece of undigested food near the back of the throat. You would:
• Perform a finger sweep.
208. Which of the following statements shows that the OEC technician understands how to suction patients properly in a field setting?
• When there is active bleeding in the mouth, I may need to use gravity and suction concurrently.
209. To clear an airway of fluid and debris:
• Insert the tip of the suction catheter only as far as you can see.
210. You have opened and cleared your unresponsive patients airway, and there is no evidence of head or spinal injury. While waiting for other equipment or personnel, the easiest way to maintain this state and ensure adequate breathing is to put this patient into the recovery position. Which of the following best describes this position?
• Place the patient onto his left side.
211. You have been asked to teach a new patroller how to put a patient in the recovery position. Which of the following statements about the recovery position would be correct?
• Turn the patient on his side and flex his upper leg so that it anchors him on his side.
212. By placing an oropharyngeal airway in a patient, you have:
• Kept the tongue from occluding the airway.
213. Which of the following statements indicates that the speaker understands how and when to use an oropharyngeal airway?
• I must watch the patient for vomiting or gagging even with the proper insertion.
214. For which of the following patients is placement of an oropharyngeal airway indicated?
• An unresponsive patient who has neither a gag reflex nor a cough reflex
215. Which of the following statements indicates that the speaker knows how to properly size an oropharyngeal airway before its placement?
• The length of the airway should approximate the distance from the corner of the mouth to the angle of the jaw.
216. You have been ventilating the patient with an oropharyngeal airway and bag-valve mask. Suddenly the patient regains consciousness and starts to gag. Your immediate action should be to:
• Remove the airway.
217. You appropriately size a nasopharyngeal airway by measuring the:
• Distance from the patients nose to the earlobe.
218. You are observing a rescuer insert an oropharyngeal airway into the airway of a 36-year-old male who has overdosed. Which one of the following observations indicates correct technique?
• The rescuer inserts the airway into the patients mouth with the tip pointed towards the roof of the mouth and then turns it 180 degrees once it is halfway in the mouth.
219. You recognize that an oropharyngeal airway has been appropriately inserted when:
• Air moves freely in and out of the airway.
220. If a patient gags while you are inserting an oropharyngeal airway, you should remove the airway and:
• Have suction ready.
221. You hear a candidate OEC technician explaining the use of a barrier device to another candidate. Which of the following statements would indicate that the candidate fully understands its purpose or how to use it?
• When ventilating a patient with a barrier device, watch for the rise and fall of the patients chest.
222. Which of the following statements shows an understanding by the speaker of the use of a pocket mask?
• A pocket mask must be securely placed over the patients mouth and nose.
223. If while checking a size D oxygen tank you notice that the reading on the pressure regulator is 1000 psi, you should recognize that the tank is:
• Half-full.
224. You are checking equipment at the beginning of your shift. The D size oxygen cylinder measures 1000 psi. You calculate that if you needed to use this tank at 15 LPM it would last:
• 8.5 minutes
225. To have a maximum amount of oxygen available for an incident with many patients, you would stock which size of tank?
• M
226. What is the most common oxygen delivery device used by OEC technicians?
• A nonrebreather mask
227. Which of the following statements shows that the speaker understands how to properly store an oxygen tank after use?
• Ill place the oxygen tank in its protective case until we need to use it again.
228. You have been asked to put a patient on a nonrebreather mask at 15 LPM. After you have finished attaching the regulator to the oxygen cylinder, your next step is to:
• Turn the valve stem on the top of the cylinder using a special oxygen wrench or key.
229. Which of the following occurs during the proper application and use of a nonrebreather mask?
• The oxygen reservoir should be inflated prior to application of the mask.
230. Despite coaching and explaining the benefits of a nonrebreather face mask, a female patient with chest pain panics and states that she cannot tolerate the mask over her face. The more that she panics, the worse the chest pain becomes. Your best course of action is to:
• Disconnect the nonrebreather and replace it with a nasal cannula.
231. Which of the following statements describes the correct positioning of a BVM on an adult patients face?
• The narrow part of the mask is over the bridge of the nose, and the bottom part is in the groove between the lower lip and chin.
232. When monitoring a patient receiving oxygen through a nasal cannula, which of the following observations warrants immediate intervention?
• An oxygen flow rate of 15 LPM
233. Shock can best be described as:
• Inadequate tissue perfusion in which cells do not receive sufficient amounts of oxygen and nutrients.
234. A patient in early onset of shock informs you that he has had severe diarrhea and vomiting over the past four days. Given this history, you would recognize the pathophysiology of the shock is probably related to:
• The loss of plasma volume.
235. Which of the following lists of assessment findings indicates that a trauma patient may be in compensated shock?
• Anxious, tachycardia, normal BP, pale and cool skin
236. You are reassessing a patient who has sustained blunt trauma to the chest. Which one of the following reassessment findings best indicates that the patient is deteriorating and is in the decompensating phase of shock?
• Blood pressure of 88/50
237. In decompensated shock, the failure of body systems despite the bodys attempt to oxygenate vital organs becomes apparent as:
• Increased pulse rate and decreased blood pressure.
238. A patient with severe gastrointestinal bleeding is in what type of shock?
• Hemorrhagic
239. The four major types of shock are:
• Hypovolemic, cardiogenic, obstructive, and distributive.
240. Which of the following statements made by a patients family member would lead you to suspect that the patient is suffering from hypovolemic shock?
• He cannot stop throwing up.
241. A patient in shock with abdominal pain indicates he noticed lots of blood in the toilet after having a bowel movement this morning. You would recognize the possibility of what type of shock?
• Hypovolemic
242. A patient who sustained blunt trauma to the abdominal and pelvic areas in a very serious collision is probably in which type of shock?
• Hypovolemic
243. Which of the following conditions is the most probable cause of cardiogenic shock?
• Myocardial infarction
244. The underlying cause of distributive shock is:
• Dilation of blood vessels.
245. Septic shock is caused by:
• An infection.
246. Which of the following conditions could be responsible for causing obstructive shock?
• Pulmonary embolism
247. You approach a patient and notice a large amount of blood on the ground. The patient appears confused, pale, and diaphoretic. Which one of the following should you do first?
• Ensure that the patients airway is open and clear.
248. When performing a primary assessment, one of the first signs that the body may be in shock is:
• Tachycardia.
249. Which of the following statements best indicates that the speaker understands the role of caring for a patient in shock in a prehospital setting?
• Because shock is best treated in the hospital, one should provide care to correct problems affecting ABCDs and then rapidly transport the patient.
250. A 56-year-old female has struck a tree. Your assessment reveals gurgling respirations, rapid breathing, and cool, diaphoretic skin. You also observe bruising to the chest and abdomen. Which one of the following should be your first concern?
• Open and maintain her airway.
251. You get a call for a serious injury on the expert trail. As you are skiing up to the patient, your first priority is:
• Assessing the scene for rescuer safety.
252. When assessing a patient with a possible stroke, the OEC technician should first work on:
• Recognizing the signs of a stroke.
253. Glucose:
• Is an energy source for brain cells and other cells in the body.
254. In the mnemonic AEIOU-TIPS, the A stands for:
• Alcohol and acidosis.
255. In the mnemonic AEIOU-TIPS, the E stands for:
• Epilepsy, environment, and electrolyte imbalance.
256. The signs and symptoms of a transient ischemic attack (TIA):
• Are temporary and resolve within 24 hours.
257. In the mnemonic AEIOU-TIPS, the O stands for:
• Oxygen and overdose.
258. In the mnemonic AEIOU-TIPS, the U stands for:
• Uremia (kidney failure).
259. In the mnemonic AEIOU-TIPS, the T stands for:
• Trauma and tumors.
260. In the mnemonic AEIOU-TIPS, the first and second I stand for:
• Insulin and infection.
261. In the mnemonic AEIOU-TIPS, the P stands for:
• Poisoning and psychiatric conditions.
262. In the mnemonic AEIOU-TIPS, the S stands for:
• Seizure, stroke, and syncope.
263. Which of the following statements concerning the causation of seizures is correct?
• A seizure occurs when there is an electrical disturbance in the brain.
264. You are called to the lodge to assess a 3-year-old boy. The parents state that their son was playing with his brother and blanked out and stared off into space for several seconds. They deny any convulsing-like movement. Based on this description, you would suspect what type of seizure?
• An absence (petit mal) seizure
265. You have been called to the lodge for a behavioral emergency. When you arrive, you find a male in his forties sitting up against a wall. He is confused and incontinent. Bystanders state that he suddenly fell to the ground and was grunting and shaking. From this description, what should you suspect first that the man has had?
• A generalized seizure
266. Some patients experience a premonition indicating a seizure is about to happen known as a(n):
• Aura.
267. Which of the following statements indicates that the speaker understands the danger posed by status epilepticus?
• The longer the seizure continues, the greater the likelihood of permanent brain damage.
268. The primary problem in Type II diabetes is that:
• Cells exhibit resistance to insulin preventing glucose from entering the cells.
269. Which of the following actions may result in hypoglycemia in a diabetes patient?
• Missing a meal when the patient has taken their insulin
270. Hyperglycemia:
• Has a more gradual onset than hypoglycemia.
271. Which of the following events best describes an ischemic stroke?
• A blood clot has obstructed a blood vessel in the brain.
272. Which of the following statements indicates that the speaker understands hemorrhagic strokes?
• They occur when a blood vessel in the brain ruptures.
273. Which of the following questions is critical for establishing a window of treatment for a patient who is weak and dizzy and may have suffered a stroke?
• What time did the weakness and dizziness start?
274. After a delayed response, you arrive in the lodge and find a 62-year-old female is still seizing. Which one of the following should you do first?
• Assess her airway and breathing.
275. When performing a secondary assessment on a confused patient, which of the following signs is most suggestive of a seizure?
• A bitten tongue
276. You have assessed a patient and found a patent airway, adequate breathing, and a strong radial pulse. The patient is having no difficulty speaking and no facial droop but exhibits a slight left arm drift. You should interpret these findings as:
• Suggestive that a stroke may be occurring.
277. Which of the following statements shows that the speaker has an accurate understanding of assessment findings related to stroke?
• If one arm is weak, a stroke should be suspected.
278. You have been called for a 63-year-old woman with slurred speech and right arm weakness. On arrival, the patient informs you that the slurred speech and weakness have resolved. Since your assessment reveals no deficits, you would tell the patient:
• You really need to be evaluated in the hospital. You could be having a stroke.
279. Which of the following instructions to a possible stroke patient describes the proper procedure for assessing an arm drift?
• Hold your arms straight out in front of you with your palms facing up with your eyes closed.
280. Which letter in FAST-ED involves the eyes?
• E
281. Your assessment reveals a drooling 48-year-old male to have gurgling speech and left arm paralysis. Which of the following would be your priority?
• Suction the patients airway.
282. You have been called for a seizure emergency. You find an adult female actively seizing with a bystander attempting to place a spoon between her teeth. The bystander tells you that he is trying to keep the person from biting her tongue. Which one of the following would be an appropriate response?
• Ask the bystander to discontinue his efforts and then turn the patient onto her side to facilitate the drainage of oral secretions.
283. A young boy tells you that his twin brother suffers from seizures and asks what he can do if he sees his brother convulsing. You should give him which one of the following instructions?
• Move any moveable objects and furniture away from him.
284. As a general rule, a seizing patient should be transported in what position?
• On their side
285. The passage of oxygen and carbon dioxide through the walls of the alveolae occurs via:
• Diffusion.
286. Which of the following lists best represents the correct sequence for the passage of air into the lungs once it passes the pharynx?
• Larynx, trachea, bronchi, alveoli
287. When the diaphragm and intercostal muscles relax, which of the following events occurs?
• Exhalation
288. The most important muscle of respiration is the:
• Diaphragm.
289. Which of the following actions causes an individual to inhale?
• The diaphragm contracts and flattens.
290. You are documenting your assessment of the patient who just left in an ambulance. The patient told you that he felt short of breath. You would document this as:
• Dyspnea.
291. The normal range for respiratory rate in children is:
• 15-30 breaths per minute.
292. You receive a call that there is an infant in the lodge who seems to be having difficulty breathing. As you are approaching the scene with a new candidate patroller, you ask him what the normal respiratory rate is for an infant. The candidate would be correct if he told you the rate should be:
• 20-60 breaths per minute.
293. Which of the following statements about respiratory accessory muscles is not correct?
• They decrease negative internal chest pressure to draw more air into the lungs.
294. In an adult, the most common cause of airway obstruction is:
• The tongue.
295. A 74-year-old male with a history of chronic emphysema is complaining of sudden onset of shortness of breath. He appears to be in acute respiratory distress. Physical exam of the anterior chest wall reveals crepitus. Based on the patients history and physical exam, you would suspect which of the following conditions?
• Spontaneous pneumothorax
296. Hyperventilation syndrome is a common psychological condition that:
• Results in abnormally low blood carbon dioxide levels.
297. A tachypneic patient is breathing:
• More rapidly than normal.
298. During a scene size-up, which of the following observations most strongly suggests that your adult patient is suffering from an acute respiratory emergency?
• The patient is in the tripod position.
299. You and your OEC candidate are evaluating a 67-year-old patient with respiratory distress. You tell the candidate to document that the patient complains of dyspnea. The candidate asks you what dyspnea means. You explain that dyspnea is:
• A subjective term that means difficulty breathing.
300. Which of the following signs would you recognize as an early sign of respiratory distress in a 7-year-old boy who complains of difficulty breathing?
• Nasal flaring
301. You are having a hard time getting the medical history of a patient who is short of breath. Which one of the following pieces of information would lead you to believe that the patient has a history of a chronic lung disease such as chronic emphysema?
• Her chest is barrel shaped.
302. You are assessing a 24-year-old skier who is anxious, wheezing, and complaining of shortness of breath. She tells you that this sometimes happens when she is exercising. Her respirations are rapid and shallow. Based on this information, you suspect the patient is suffering from:
• Asthma.
303. Which of the following respiratory rates may signify significant respiratory issues in an adult?
• Fewer than 8 respirations per minute
304. You are assessing a 34-year-old man who is complaining of not feeling well. He is alert and pale. His respirations are regular at 20 per minute. Based on your training, you recognize that his respirations are:
• Normal for an adult.
305. You are called to assist a 16-year-old female who is reportedly having trouble breathing. Your assessment reveals an anxious female with rapid respirations who is complaining of pins and needles around her lips. Based on your training you suspect that:
• These findings will disappear if she slows her rate of breathing because she is most likely hyperventilating.
306. Several abnormal lung sounds can be helpful in understanding what condition may be affecting a patient. One such sound is wheezing. Which of the following statements about wheezing is false?
• It can be heard only with a stethoscope.
307. A patient informs you that any time she uses a particular soap, she experiences a mild allergic reaction. You recognize that:
• The soap contains an allergen.
308. Which of the following statements indicates that the speaker understands allergic reactions and anaphylaxis?
• Anaphylaxis is a severe allergic reaction that will lead to death without emergency care.
309. You are called to the lodge to help a patient who has developed an allergic reaction after taking an antibiotic medication. You would describe this route of exposure as:
• Ingestion.
310. An acquired, abnormal immune response to a substance that does not normally cause an allergic reaction is known as:
• An allergy.
311. A patient can have an allergic reaction caused by intramuscular administration of a medication. This method by which an allergen enters the body is called:
• Injection.
312. The overreaction of the immune system to a substance that is otherwise harmless is known as:
• Hypersensitivity.
313. Which of the following statements concerning allergies is true?
• An allergic reaction to a substance can occur minutes, or hours, after exposure to the substance.
314. Which of the following statements from a patient suggests he may be having a mild allergic reaction?
• I have a rash on my face and neck.
315. During a primary assessment, which of the following findings would most likely lead you to suspect an allergic reaction?
• Hives on the face and neck
316. The patient has swollen lips and hives. Which of the following scene size-up observations supports your suspicion that the patient is having an allergic reaction?
• He has a new container of antibiotics with only two pills missing.
317. Which of the following foods most commonly causes an allergic reaction?
• Peanuts
318. The most severe form of an allergic reaction is called:
• Anaphylaxis.
319. A patient who is responsive only to painful stimuli is covered with hives. He has labored respirations and a weak and tachycardic pulse. His vital signs are pulse 128, respirations 24, and blood pressure 80/50 mmHg. Given this presentation, you would suspect:
• An anaphylactic reaction.
320. When the respiratory effects of an allergic reaction are obvious within a few seconds to minutes:
• The reaction tends to be more severe.
321. Which of the following findings is present in anaphylaxis but is not a sign of a mild allergic reaction?
• Hypotension
322. Your patient is having an allergic reaction and needs to use his epinephrine auto-injector. He is concerned that he does not recall the correct way to do it. Which of the following things would you tell him?
• Press the injector onto the outer thigh and push the tip against the thigh until the needle deploys."
323. You are staffing a first-aid booth at a mountain bike event. A mother approaches you with her 15-year-old son and informs you that her son is severely allergic to bees. Although he hasnt been stung, she is concerned about the auto-injector he carries. The mother states that it was prescribed when her son was 13 years old. She shows you the 0.15 mg auto-injector, and you note that it expired about 12 months ago. Which of the following statements would be most appropriate for you to make?
• Call your doctors office for a new prescription because this auto-injector is out of date.
324. When administering epinephrine to an infant or child, the injector should be placed at what location?
• The lateral thigh
325. The walls of the capillaries are only one cell thick. This allows the exchange of nutrients and oxygen and waste products. The capillaries serve as the bridge between:
• Arterioles and venules.
326. Which of the following statements concerning the flow of blood to and through the right side of the heart is correct?
• Poorly oxygenated blood reaches the right atrium from the venae cavae, is pumped through the tricuspid valve into the right ventricle, and is then pumped through the pulmonic valve and on to the lungs through the pulmonary arteries.
327. The left side of the heart:
• Pumps oxygenated blood to the body.
328. After jogging with your friend for about 30 minutes, she stops and asks why your heart rate increases and your heart feels like it is pumping harder after you exercise for a while. Based on your understanding of the cardiovascular system, your best response would be which of the following statements?
• Exercise makes your body need more nutrients and oxygen. Your heart pumps harder and faster to deliver more blood containing those things to your muscles.
329. Which of the following statements concerning arteries is true?
• They carry blood away from the heart.
330. Which of the following statements concerning the cardiovascular system is true?
• An electrical impulse that starts in the right atrium stimulates contraction of the heart muscle.
331. The largest artery in the body is the:
• Aorta.
332. Which of the following statements about cardiovascular disease is false?
• Most cases of cardiovascular disease are congenital and could be prevented with improved prenatal care.
333. Your neighbor tells you that he is concerned about his 70-year-old wife. She is at home recovering from a heart attack that occurred about 3 weeks ago. He explains that over the last few days, her ankles and lower legs have been gradually swelling and are painful to touch. Today, she seems to be a little short of breath. You recommend that your neighbor seek immediate medical attention because these findings are suggestive of:
• Congestive heart failure.
334. Which of the following statements indicates that the speaker has an understanding of chest pain and acute myocardial infarction?
• Some patients experiencing acute myocardial infarction do not have actual chest pain. Instead, they may experience a sensation of severe heartburn.
335. A patient tells you that he has been diagnosed with coronary artery disease. You recognize this diagnosis to mean that the patient has:
• Narrowing of the small arteries that supply blood and oxygen to the heart.
336. A patient with chest pain informs you that he has a bad heart. You think he may have congestive heart failure. When performing your secondary assessment, which of the following signs or symptoms does not indicate that the patient is in congestive heart failure?
• Bradycardia
337. You are out shopping when a middle-aged man suddenly falls to the ground. You quickly run to assist him and note that he is not breathing and does not have a carotid pulse. You begin CPR because you know that prompt intervention with an AED is needed. Which of the following statements is not a reason that prompt intervention with an AED is needed?
• Asystole, which can accompany an acute MI, is often reversed by the use of an AED.
338. How do most problems with heart valves originate?
• Congenitally
339. You are called to assist a 62-year-old female who is complaining of fatigue, jaw discomfort, and excessive sweating. When asked, she denies having chest pain or shortness of breath. She tells you that she has a history of hypertension and takes a blood pressure pill. She also has nitroglycerin. Which statement is inappropriate for this event?
• She is not short of breath, so we don't need to transport her.
340. The only reason to delay transport in case of a cardiac emergency is:
• To use an AED.
341. Aspirin may be used in a patient who has chest pain suggestive of a heart attack because aspirin may:
• Decrease the ability of platelets to form clots.
342. You are caring for a patient with chest pain who has just taken his nitroglycerin. The patients chest pain has been alleviated. You understand that nitroglycerin is effective in treating chest pain because it:
• Dilates the arteries and veins, which decreases the workload on the heart.
343. Three commonly used cardiac medications are:
• Nitroglycerine, Coumadin, and Lasix.
344. You have just arrived with an AED at a scene where fellow OEC technicians are treating a patient in cardiac arrest. EMS has been contacted and is about 1 minute away. Which of the following actions should you take?
• Apply and use the AED.
345. Which of the following signs or symptoms is suggestive of right-sided congestive heart failure?
• Edema in the legs and feet
346. An alert and oriented 65-year-old woman presents with shortness of breath, noisy respirations, jugular vein distention, and edema in her feet and ankles. Her pulse is 132, her respirations are 24, and her blood pressure is 86/68. Based on these findings, you would suspect that the patient:
• Is experiencing congestive heart failure.
347. Which of the following statements made by a patient would cause you to suspect an abdominal aortic aneurysm?
• I have really bad belly pain that I also feel in my back.
348. You are assessing a 68-year-old man with a history of abdominal aortic aneurysm. How far might pain be radiating?
• Groin
349. Hypotension, distended neck veins, and muffled or distant heart tones are hallmark signs of:
• Pericardial tamponade.
350. A 62-year-old man presents with a complaint of abdominal pain radiating to his groin. He also complains of dizziness. On examination, his abdomen is tender, and you feel a large pulsating mass. His signs and symptoms are suggestive of:
• An abdominal aortic aneurysm.
351. You are assessing a 38-year-old woman who is complaining of the sudden onset of chest pain and shortness of breath. She describes the pain as sharp and states that it increases when she takes a deep breath. You note that she is breathing rapidly. Her BP is 130/82, and her pulse is 100. Based on her presentation, you suspect that she may have:
• Pulmonary embolism.
352. When performing CPR, your philosophy should be:
• Push hard, push fast.
353. Which of the following is not a reason to stop CPR?
• It has been 25 minutes, so brain damage has occurred.
354. Blood in the right ventricle is pumped into the:
• Pulmonary artery.
355. You have been called to treat a 47-year-old man complaining of chest pain. He is alert, oriented, and complaining of pain in his chest that came on suddenly about 15 minutes ago. His skin is sweaty and cool. Which of the following actions should you take first?
• Apply oxygen at 15 LPM through a nonrebreather mask.
356. You are in the first-aid area assisting a patient who just arrived with chest pain. He relates a history of angina that usually resolves with nitroglycerin. The patient states that the pain started when he was skiing through the bumps. Over the last 20 minutes, he has taken three nitroglycerin tablets and rested, but the pain has not gone away. Your next action would be to:
• Place him on high-flow oxygen and call for transfer to medical facility.
357. When assisting a patient to take a nitroglycerin tablet, it is:
• Placed under the tongue.
358. Before nitroglycerin is administered, you should ensure that:
• Nitroglycerin is in fact prescribed for the patient.
359. Which of the following medications, if taken in the last 24 hours, would be a contraindication to administering nitroglycerin?
• Viagra
360. You are assisting a 67-year-old patient with chest pain. He has taken two nitroglycerin tablets, but his pain remains a 7 out of 10. He is diaphoretic; his respirations are 18 per minute and adequate; his pulse is 72; and his blood pressure is 80/62 mmHg. He is on a nonrebreather mask at 15L per minute. You would:
• Reassess his vital signs every 5 minutes and transfer him by EMS care.
361. Which of the following actions is most important before assisting a patient to take prescribed nitroglycerin?
• Evaluate the patients blood pressure.
362. When a patient can point to an exact location of pain over their abdomen, this is known as _________________ pain.
• Parietal
363. When a patient has generalized abdominal pain and can only describe a larger area that is painful, it is known as _____________ pain.
• Visceral
364. Which one of the following organs is not located in the abdomen?
• Kidney
365. Lower abdominal pain could indicate which of the following conditions?
• A urinary tract infection
366. A 28-year old healthy male presents with recent signs and symptoms of nausea and vomiting but very little abdominal pain. Which one of the following is your best course of action?
• The patient has indigestion; suggest that he follow up with his physician if the symptoms continue.
367. Which one of the following statements regarding an acute abdomen is true?
• OEC technicians should be able to make a diagnosis that the patient is ill enough to warrant transport to a hospital.
368. Which of the following statements regarding assessment of the abdomen is false?
• Assessment of the abdominal organs is straightforward because the abdominal organs are specifically located and easily palpated.
369. Colic is often caused by which one of the following?
• Obstruction and distention of a hallow organ
370. What is typically the most comfortable position for a responsive patient who is suffering from abdominal pain?
• Supine with knees slightly flexed
371. When dealing with a 61-year-old female patient with acute lower right abdominal pain, which quadrant should be examined first?
• Left upper
372. The function of the uterus in pregnancy is to:
• Provide an environment for the ovum to develop into a fetus.
373. Which of the following statements concerning the functions of the vagina is true?
• The vagina serves as a passageway for the fetus during delivery and an outlet for blood during menstruation.
374. Which of the following statements about the umbilical cord is true?
• It transmits nourishment from the placenta to the fetus.
375. Which of the following findings should be of greatest concern when assessing a 33-year-old woman who is 8 months pregnant?
• Vaginal bleeding and abdominal pain
376. The most common cause of fetal death due to trauma to a pregnant womans abdomen is:
• Abruptio placentae.
377. In abruptio placentae, the greatest threat to the baby is:
• Hypoxia.
378. Hypotension can occur after 20 weeks of gestation when a pregnant woman is in a supine position. Which of the following actions by an OEC technician will not prevent hypotension?
• Elevating her feet 6-12 inches
379. The second stage of labor ends with:
• Delivery of the baby.
380. At what point during a delivery should you consider using suction to clear the airway of the baby?
• After the babys head has been delivered but before the torso is out of the vaginal opening and if there is obvious obstruction to spontaneous breathing
381. A heavy-set snowboarder was critically injured when he crashed into a tree on a dimly lit trail. It was reported that he appeared to be out of control while traveling at a high rate of speed down the expert trail. Which of the following factors had the greatest impact on the extent of his injuries?
• The speed the skier was going
382. Which of the following best defines mechanism of injury?
• The method that an injury occurs
383. Which of the following factors does not directly relate to the severity of bodily injury?
• The size of the injured person
384. A patient involved in an altercation was struck in the ribs with a baseball bat. Your assessment reveals intact skin with significant bruising to the right lateral chest. When palpating this area, you note instability and crepitus to the rib cage. An OEC technician would recognize a:
• Chest injury caused by blunt trauma.
385. A 49-year-old man has been stabbed in the lower right chest. After assessing him, you suspect that the knife punctured the lung and is causing internal bleeding. In this situation, the mechanism of injury would be:
• A penetrating injury.
386. Your friends are discussing a hunting accident they heard about on the news. A hunter was shot with a high-powered rifle, and the report indicated that he had damage to several internal organs. From your knowledge of mechanisms of injury, you can tell your friends that this type of injury is called a:
• High-velocity penetrating injury.
387. Your patient has been stabbed in the chest with a ski pole. The mechanism of injury is:
• Penetrating.
388. Which of the following terms is not a mechanism of injury?
• Kidney stone
389. High-velocity injuries and low-velocity injuries are two types of which mechanism of injury?
• Penetrating injury
390. A high-velocity penetrating injury is usually the result of:
• The impact of a bullet from a high-powered rifle.
391. Tissue damage caused by low-velocity injuries is usually:
• Limited to the path of the object.
392. You are called to the vehicle maintenance department on the mountain, where a fairly large explosion has occurred. The first injured person you talk to tells you that he is having excruciating ear pain. You recognize that his ear pain may be caused by:
• A blast injury.
393. Dislocating a shoulder after catching a ski pole on a tree branch is an example of an injury caused by which MOI?
• Rotational injury
394. Your patient is a skier who tells you that she caught an edge and twisted her leg. You later find out that the skier suffered from a fractured tibia and fibula. This injury was the result of which mechanism?
• Rotational
395. A rock climber falls when several boulders give way. His arm is pinned between two boulders. This MOI is best described as:
• Crush.
396. Your patient has been involved in a backcountry hiking accident and has multi-system trauma. It is most important to consider which of the following when making your evacuation plans.
• Location of the nearest trauma center
397. Trauma centers have five levels, which are based on:
• Specialization, complexity, and availability of care.
398. Trauma centers are specifically designed to:
• Manage patients with multi-system trauma.
399. Assessment and management of the trauma patient should include:
• Gathering information about the mechanism of injury.
400. A 23-year-old skier falls about 20 feet from the chairlift. Aside from his complaints of soreness, you dont see any obvious injuries. The skier states that as long as his legs arent broken, hes going to continue skiing. Which of the following statements would be your best response to him?
• From a fall of that height, you may have some injuries. You really should be examined.
401. You arrive alone at a scene at which a 16-year-old skier has collided with a tree. He is unresponsive and has blood flowing from his ears and nose. He was not wearing a helmet. Which of the following lists reflects the most appropriate care of this patient?
• Primary assessment, airway management, immobilization, transport as soon as possible
402. The term golden hour refers to the:
• Time period when patient survival rates may be enhanced if critical injuries are identified and managed.
403. Which of the following lists correctly names the layers of the skin and immediately underlying tissue?
• Subcutaneous, epidermis, dermis
404. What is the function of subcutaneous fat?
• It insulates the body and stores energy.
405. A patient has a laceration on his arm from a table saw. Assessment reveals dark red blood flowing slowly from the wound. You would recognize this type of bleeding as:
• Venous bleeding.
406. You are responding to a call to aid a patient who has cut himself with a knife. Reportedly, arterial bleeding is involved. Given this information, which of the following findings do you expect?
• Bright-red blood that is spurting with each beat of the heart
407. The skin performs which of the following functions?
• Regulation of temperature
408. Which of the following injuries is most likely to result in capillary bleeding only?
• An abrasion
409. You are having a difficult time controlling bleeding from a small skin avulsion on a patients ankle. Which of the following statements made by the patient best explains why the bleeding has been so difficult to control?
• I take Coumadin (an anticoagulant) for my irregular heartbeat.
410. During your secondary assessment, a patient with a stab wound to the abdomen tells you that he is taking Coumadin, a blood thinner. Which of the following implications would taking this medication have for this patient?
• The medication could cause the patient to bleed more profusely.
411. Assessment of a patient who fell reveals bruising to her right buttock. Her skin is intact, and she complains of tenderness in the area when you palpate it. You would recognize which of the following types of injury?
• A contusion
412. When assessing a patient, you note a bruise on the chest. Another word for documenting a bruise is:
• Ecchymosis.
413. Which of the following phrases best describes a closed-tissue injury?
• A wound in which underlying tissues are damaged, but the overlying skin remains intact
414. Which of the following assessment findings best indicates that a patients injury should be classified as an abrasion?
• Red scrapes involving the outermost layer of skin only
415. A 49-year-old man was climbing on rocks when he fell backward to the ground. He presents with a two-inch linear wound on the top of his head. Bleeding has been controlled. You would document this type of injury as a(n):
• Laceration.
416. You respond to a patient who has an avulsion on the left arm. When you assess this patient, you would expect to find:
• A loose flap of torn skin on the left arm.
417. A patient has been stabbed in the chest with a 6-inch-long ice pick. When you assess the wound, you find it to be very small and insignificant in appearance, with minimal bleeding. Which of the following assumptions can you correctly make about this injury?
• Damage to the underlying structures is likely.
418. Mechanical tattooing is defined as:
• The introduction of foreign debris such as dirt, gunpowder, or small rocks into the skin.
419. Thermal burns caused by the skin directly contacting hot water or hot steam:
• Is the most common type of burn.
420. Which of the following substances cause the most serious chemical burns?
• An acid with a pH of 1
421. Exposure to direct or alternating electrical current produces severe external and internal burn injuries because:
• The skin is a poor conductor with a high resistance, and blood vessels are good conductors with low resistance.
422. When managing electrical burns, OEC technicians should always:
• Assess for signs and symptoms of internal injuries.
423. Which of the following statements concerning a patient who received electrical burns to his hand after grabbing a live wire is true?
• The extent of tissue damage may be much greater than it appears on the surface.
424. In a hospital accident, what might cause radiation burns?
• X-ray machine
425. Severe external bleeding should be controlled during what phase of a patient assessment?
• The primary assessment
426. When managing a patient with soft-tissue injuries, it is essential that you:
• Use the appropriate personal protective equipment.
427. Which of the following sentences best describes a partial-thickness burn?
• The skin is red and moist, and blisters have formed.
428. A burn extending through the epidermis and dermis layers into the subcutaneous layer would be classified as a:
• Full-thickness burn (third- or fourth-degree burn).
429. Which degree of burn causes damage to bone?
• Fourth
430. Your patient is a 40-year-old man who was burned when he spilled gasoline on his pants while he was standing near the pilot light of his hot-water heater. He has partial thickness burns from his feet to just above his knees, circumferentially around both legs. According to the Rule of Nines, the man burned ____ of his body surface.
• 18%
431. Hoarseness or voice changes in a burn patient should alert an OEC technician that the:
• Patient most likely is experiencing an inhalation injury causing swelling to the airway, a true emergency.
432. In the Rule of Nines, the surface of the left leg is about what percentage of the body surface?
• 18%
433. Thermal burns result from:
• Direct contact between a heat source and the skin.
434. You arrive at a scene at which a 16-year-old girl tells you she has cut her wrist with a knife. She appears to be pale, and blood is spurting from the cut. Which of the following actions should you take first after making the scene safe?
• Apply direct pressure to the cut
435. You are treating a patient who has cut herself while working in the lodge kitchen. She has a jagged laceration on her left forearm that is bleeding steadily and heavily. While attempting to control the bleeding, you should first:
• Place a sterile dressing over the site and hold steady pressure on the dressing with your gloved hand.
436. You have just arrived on scene, where a young girl has pushed her arm through a plate glass window. She is responsive and has a patent airway. Bright red blood is spurting from a large laceration on her right upper arm. When your partner tells you to apply direct pressure to the laceration to prevent the further blood loss, you would:
• Put gloves on before applying direct pressure over the injury site.
437. Which of the following statements about pressure dressings and bandages is true?
• Bandages should be applied in a distal-to-proximal manner.
438. For which of the following injuries may elevation be used to control bleeding?
• For no injuries, as elevation has not been shown to help control bleeding
439. Which of the following actions should you take next when direct pressure and packing the wound have failed to control arterial bleeding on a patients lower leg?
• Apply a tourniquet below the knee.
440. Which of the following statements indicates that the speaker understands the application of a tourniquet?
• A tourniquet should be applied tightly enough so that arterial blood flow distal to the tourniquet is completely stopped.
441. Which of the following statements about the use of a tourniquet is true?
• It should be placed as distal as possible but at least several inches proximal to the wound.
442. A wire would be a poor choice for making a tourniquet since a tourniquet should be at least how wide?
• 2 inches
443. A young boy was out riding his bike in the neighborhood when he fell onto a stick protruding from the ground. The stick impaled him in the neck. The boy immediately pulled the stick out. Assessment reveals a gaping wound to the boys right neck. Which of the following types of dressing would you use on this injury?
• An occlusive dressing that is taped on all sides
444. Which of the following statements indicates that the speaker understands bandaging?
• Bandaging material is used to secure a dressing in place.
445. You have applied a pressure dressing to the calf of a woman who suffered a deep laceration from a piece of broken glass. Which of the following actions should you take next?
• Check CMS in the patients foot
446. Which of the following statements indicates that the speaker understands dressing and bandaging a wound?
• Leave the tips of the fingers or toes exposed when bandaging an arm or a leg so you can continue to assess CMS.
447. The most effective method for treating a contusion is to apply a(n):
• Ice pack.
448. You are assessing a patient who received multiple injuries in a fight. Which of the following injuries would present the greatest danger for infection?
• An abrasion on the right side of the face
449. Which of the following instructions is appropriate for helping a new patroller care for a patient with a very large abrasion on his arm?
• Wash the gross contamination off, pouring on sterile saline, put on a sterile dressing, and recommend the patient go to the hospital for further care.
450. A patient has had part of his right thumb amputated in an accident. Friends have retrieved the thumb and wrapped it in a towel. When you arrive, you would demonstrate appropriate handling of the amputated part by:
• Wrapping the thumb in a moist sterile dressing, placing it in a sterile bag, and keeping it cool.
451. Which of the following statements about amputations is true?
• All amputated body parts should be preserved and sent to the hospital with the patient.
452. Which of the following statements about impaled objects not affecting the airway is true?
• The impaled object should be left in place during transport.
453. Which of the following statements about a high-pressure injection injury is false?
• The presenting wound is typically large.
454. Which of the following statements about an impaled object is true?
• The primary treatment goal in the field is to stabilize the object in the position found.
455. Your patient is a 25-year-old man who has a reddened area with blisters across the palm of his hand after grabbing the handle of a very hot iron skillet. Which of the following actions must be avoided in the prehospital management of this wound?
• Applying an antibiotic ointment
456. For the OEC technician, the first priority in managing a patient with a thermal burn is:
• Stopping the burning process to prevent further injury.
457. Your patient is a 35-year-old woman who spilled a cup of hot coffee on herself two hours ago. An area on her right thigh that is about twice the size of the palm of her hand is red and painful but has no blisters. Which of the following actions would be appropriate for an OEC technician to take?
• Apply a dry sterile dressing
458. In treating a burn patient, you should cool and irrigate the burned tissue with:
• Room-temperature tap water
459. Your patient is a 10-year-old boy who was exposed to a dry chemical powder and is complaining of severe pain at the site of contact on both of his hands. There is no decontamination shower on site. Which of the following actions would be the best way to manage this situation?
• Brush away as much of the powder as possible and then have the patient hold his hands under running water from a faucet or garden hose.
460. Chemical burns result from exposure to:
• Caustic substances.
461. For the OEC technician, the first priority in managing a patient with an electrical burn is:
• Ensuring the power source has been turned off.
462. When treating a patient burned by a dry chemical, it is important to brush away any dry residue before flushing with water because:
• Some dry chemicals are activated by water.
463. Which of the following functions is not one of the several functions of bones?
• Responding to sensory nerve stimulation to protect skin and other tissues
464. The inside layer of a joint capsule where cells make a viscous fluid for lubricating the joint is the:
• Synovium.
465. All of the following terms describe a type of bone except:
• Short.
466. The outermost part of a bone is a tough lining known as:
• The periosteum.
467. An example of a joint that permits very little movement of the bones involved is the:
• Acromioclavicular.
468. Which of the following terms does not describe a type of joint?
• Post
469. During your SAMPLE inquiry, a patient reports that he had surgery to repair a torn ligament. Based on your training, you know that a ligament is:
• A tissue that stabilizes two contiguous bone ends.
470. The type of muscle found in the forearm is:
• Skeletal muscle.
471. The structure responsible for transmitting the force of a contracting skeletal muscle to a bone is a:
• Tendon.
472. While you are putting wood into your wood stove, your hand comes in contact with the hot surface. Your body responds by pulling your hand away from the heat. How is this action accomplished?
• One or more skeletal muscles receive a signal from the brain to contract.
473. Which of the following statements about skeletal (voluntary) muscles is true?
• They generally connect to the skeletal system.
474. A fracture of a bone will cause bleeding, and a hematoma forms around the fracture site. Over the next several weeks, this hematoma organizes into a substance called:
• A callus.
475. Which of the following statements about musculoskeletal injuries is true?
• A sprain is an injury to a joint that involves the stretching or tearing of ligaments.
476. Which of the following musculoskeletal tissues does not heal well?
• Cartilage
477. A patient for whom you recently provided care stops by to thank you. He reports that he was diagnosed with a strain to his lower right leg. Which of the following structures is the primary structure affected in that injury?
• A muscle
478. Your secondary assessment of a patient who is complaining of pain in his right leg reveals an opening in the skin where the fractured tibia broke through the skin and retreated back into the leg. You would recognize this injury as a(n):
• Open fracture.
479. The finding that upon muscle contraction the active motion of a joint is reduced or lost suggests:
• A ruptured tendon.
480. Which of the following statements indicates that an OEC technician has a good understanding of what a joint dislocation is?
• A dislocation is a separation or displacement of the bones of a joint.
481. Displacement of the bones of a joint is known as:
• A dislocation.
482. The most commonly dislocated joint in the body is:
• The shoulder.
483. A 42-year-old man has fallen 25 feet while rock climbing. He hit the ground feet first and suffered open fractures to both tibias, which are protruding through the skin. When you arrive, you note the scene is safe, and the patient is responding to painful stimuli. His airway is open, his breathing is adequate, his radial pulse is strong and rapid, and your patient is alert and oriented. At this point in his care, it is a priority for you to:
• Perform a secondary survey to look for other injuries.
484. Your patient fell 20 feet onto his shoulder. Which of the following signs or symptoms indicates the highest probability that the humerus has been fractured?
• Angulation
485. Your friend tells you that he wants to try snowboarding. He is concerned about his safety and asks you which type of fracture is most common in snowboarders. You learned in your OEC training that this injury involves the:
• Radius.
486. A common injury to the carpal bones among snowboarders who fall forward on an outstretched hand is a fractured:
• Scaphoid.
487. A snowboarders fall over the toe side of the board onto an outstretched hand is known as the:
• Mousetrap.
488. Which of the following statements would you make to the parent of a 10-year-old snowboarder who took a hard fall and complains of some discomfort in a wrist that is slightly swollen but has no deformity?
• The child needs to be medically evaluated to ensure there is not a fracture.
489. The _____________ can be injured when a hip is dislocated, resulting in numbness or paralysis of a lower extremity.
• Sciatic nerve
490. A fracture that has three or more fragments is called a(n):
• Comminuted fracture.
491. The most frequent injury in skiing is a:
• Knee sprain.
492. You respond to a 9-year-old boy who was injured in the terrain park. He is complaining of pain in his right wrist. Upon examination, you note deformity and swelling of the right wrist, pinkness of the right hand, and a strong radial pulse. Which of the following actions would be part of the proper care for this boy?
• Apply a cold pack to the wrist to reduce swelling.
493. You are treating a patient who is complaining of moderate pain in the right knee. The knee is swollen, discolored, and flexed about 45 degrees. CMS is intact. How would you treat this patient?
• Splint the knee in the position found before moving the patient.
494. You are notified that a patient is coming to the first-aid room with bruising to the lumbar area of the back. Based on this statement, you would expect to find bruising in which area?
• The lower back
495. You respond to an accident at which a male patient is complaining of severe back pain. He informs you that his back pain is coming from a recent fracture of his coccyx. Based on this information, what area of the back would the pain be emanating from?
• The lowest part of the spine
496. Which of the following signs or symptoms might be evident for an injury to the autonomic nervous system?
• Elevated heart rate
497. A patient was killed immediately following a self-inflicted gunshot wound to the head. Which portion of the central nervous system was most likely damaged in order to cause the rapid death of the patient?
• The brainstem
498. For a person to make a fist, which of the following must occur?
• The central nervous system must send a message through the spinal cord and then through the peripheral nerves.
499. A 68-year-old female patient is complaining of a headache and generalized weakness. Her husband informs you that she was with a friend yesterday and struck her face on the dashboard during a motor vehicle collision. Her past medical history includes a stroke that caused right arm weakness and high blood pressure. When assessing this patient, which of the following findings should concern you the most?
• Bruising behind her left ear
500. Which of the following findings is most consistent with a skull fracture?
• Bruising behind the ear that develops several hours after the injury
501. A mechanical injury to the brain that results in a short-term and/or a long-term neurologic deficit is:
• A traumatic brain injury.
502. A male soccer player was struck in the head with a soccer ball. Players state that he was dazed for several seconds following the impact and then asked the same questions over and over. He is currently conscious and oriented to person but is confused as to place and time. He also has a reddened area on the side of his head and face. As you proceed with your assessment, his memory continues to improve. Based on these findings, you would suspect which of the following injuries?
• A concussion/TBI
503. Your patient is a young female who was thrown from her mountain bike and is now confused. Assessment findings include an open airway, adequate breathing, and a strong radial pulse. Which of the following questions would be the most important to ask her friends who were with her?
• Did she lose responsiveness?
504. You are participating in training OEC candidates and are asked about cerebral contusions. You would explain to the class that a cerebral contusion is:
• Bruising and swelling of the brain tissue.
505. A patient involved in a motor-vehicle collision has suffered the separation of a rib from a spinal vertebra. Based on the anatomy of the spine, where has this injury occurred?
• The thoracic spine
506. A young intoxicated male patient cannot move his lower extremities after diving into the shallow end of a pool and hitting the bottom with his head. Which of the following mechanisms is most likely to be responsible for this injury?
• Compression
507. While performing a primary assessment on a patient who has an isolated spinal cord injury, you note that he is in severe respiratory distress and struggling to breathe. Where should you suspect that the spinal cord injury has occurred?
• The cervical spine
508. A primary injury to the central nervous system would be caused by:
• A laceration of the spinal cord.
509. Which of the following signs or symptoms best indicates that a patient has suffered an injury to the thoracic spine?
• Tingling in the legs
510. You are assessing a 30-year-old man who has fallen about 20 feet. He is alert and oriented but states that he cannot move or feel his legs. Additionally, because his blood pressure is 82/48 mmHg, you suspect neurogenic shock. Which of the following additional assessment findings reinforces your suspicion of neurogenic shock?
• Heart rate of 44
511. Which of the following findings indicates that a patient who received a blow to the head is suffering from something other than a concussion/TBI?
• His pupils are noticeably unequal.
512. You should recognize a possible spinal column injury with no spinal cord involvement when you discover which of the following assessment findings?
• Tenderness to the thoracic spine with intact motor and sensory function in each extremity
513. A patient involved in a motor vehicle collision has a deformity to the left side of the head. The skin overlying the deformity is still intact. Based on these assessment findings, which of the following conditions would be your greatest concern?
• A possible brain injury
514. An elderly patient has fallen down a flight of stairs and is complaining of neck and back pain and weakness to both legs. Your primary assessment reveals no life threats to the airway, breathing, or circulation. Manual in-line spinal stabilization is being maintained. Which of the following actions should you take next?
• Complete a secondary assessment looking for injuries.
515. Which of the following statements about the care and treatment of a patient with a spinal injury in a prehospital setting is true?
• Prehospital care for a patient with a spinal injury involves correcting life-threatening injuries and limiting secondary injury.
516. Which description of a helmet would indicate that it needs to be removed in a prehospital setting?
• Too large
517. Which of the following signs is a characteristic sign that may be seen during a secondary assessment of patients with spinal cord injury?
• Flushed skin color below the level of the injury and impairment or absence of sensation and movement
518. Which of the following assessment findings is most indicative of increasing pressure within the skull from a closed head injury?
• Blood pressure of 192/82 mmHg
519. You are called to aid a 61-year-old man who has fallen off a second-floor lodge patio. He is unresponsive and has very slow, shallow breathing at a rate of 6 breaths per minute. When you arrive, a fellow patroller has already opened the airway and has stabilized the head. Which of the following actions should you take immediately?
• Assist ventilation with a bag-valve mask.
520. A patient with a head and midline posterior neck injury was sitting found leaning against a tree. You discover that your team does not have a long backboard, so you use what alternative?
• Whole body vacuum mattress
521. A 41-year-old male snowboarder struck his head on a metal rail. As you approach him, you note that he appears confused and has blood on the left side of his face and head and on his shirt. His breathing appears to be labored. Which of the following actions should you perform immediately?
• Apply manual in-line spinal immobilization.
522. Which of the following instructions from one OEC technician to another describes the appropriate application of a cervical spine immobilization collar?
• Keep his head in neutral position while I apply a cervical collar.
523. A properly sized cervical collar will rest on the:
• Back of the skull.
524. A 2-year-old boy fell down a flight of stairs. Which of the following instructions would you provide to other patrollers who are immobilizing the patient?
• Lets place a folded towel under his shoulders to help maintain his head alignment.
525. Which of the following statements about removing a helmet in a prehospital setting is correct?
• It is acceptable to leave a helmet on a patient if the patient has no airway or breathing problems.
526. When explaining the priorities of helmet removal to a candidate patroller, you would stress:
• Sliding the fingers under the occiput so that the head doesnt drop backward.
527. Which of the following structures works like a camera shutter to adjust the amount of light entering the eye?
• The iris
528. Which of the following statements concerning the eye socket or orbit is false?
• It is easy to injure the orbital bones.
529. You are assessing a patient who was punched in the eye. You note blood in the white part of his eye. This area of the eye is known as the:
• Sclera.
530. The part of the eye that focuses light on the retina is called the:
• Lens.
531. Light stimulates nerve endings in which of the following structures to send signals to the brain via the optic nerves?
• Retina
532. A 15-year-old girl struck in the mouth with a baseball bat has lost her front teeth and is spitting a significant amount of blood. Your primary concern in treating this patient is:
• Observing her for possible airway compromise.
533. You are on a snowmobiling trip with friends when you see members of the lead group hollering and waving their hands up ahead. When you get to the scene, you note that the lead rider apparently did not see a chain across the road, and he struck the chain with his neck so forcefully that he was thrown backward off his machine. Someone in the group is maintaining manual in-line stabilization. When you assess the injured person, which of the following signs would indicate the injury that must be addressed and managed first?
• Difficulty speaking
534. While running outside the lodge, a teenage girl tripped and fell. She hit her face and mouth, knocking one of her top front teeth from its socket. Although she is upset, you have assessed no threats to life, and the bleeding has been controlled. Your partner finds the tooth and asks you what to do with it. Which of the following responses would be best?
• Avoid touching the root of the tooth. We will gently irrigate it and place it back in its socket.
535. You are caring for a 15-year-old male with a laceration of the left ear. Which of the following actions would you not take in dressing this injury?
• Wrapping gauze loosely around the head and under the opposite axilla
536. A mother brings her 7-year-old child to the aid room and tells you that fluid is draining from the childs right ear. She tells you that the child has had a fever since this morning and was complaining of right ear pain. Based on your OEC training, you would:
• Have the boy lie on his right side to allow the fluid to drain.
537. A young boy was running through the lodge with a pencil and tripped. The pencil impaled the boys left eye and remains lodged in place. Appropriate care for the eye would include:
• Stabilizing the impaled object and providing immediate transport.
538. During an altercation, your patient was cut with a sharp knife. Your assessment reveals a laceration across the right eyelid down to the right cheek that is oozing dark red blood. It also appears that the patients eye was cut with the knife. You would:
• Cover both eyes with a sterile dressing.
539. What instructions would you give to an OEC candidate who asks how to treat a patient who has visible blood in the anterior chamber of the eye?
• Put a shield over the eye and cover both eyes before immediately transporting this patient.
540. A patient who was hit in the face with a cloud of dust while working in an industrial setting is complaining of pain and discomfort to his left eye. While performing your assessment on the eye, you note some redness of the globe but do not see any obvious foreign object. The most appropriate care you can provide to this patient would be to:
• Flush eye with copious amounts of water.
541. A young girl is complaining of eye pain after having bleach thrown in her face. When assessing her eyes, you note redness and significant tearing in the right eye. You would treat this girl by:
• Flushing the eye with sterile water.
542. You have responded to a scene at which a young man tells you he splashed a chemical into his eye. He is complaining of burning and pain in his right eye. After performing a primary assessment and finding no life-threatening conditions, you would:
• Hold the patients injured eye open and flush it with large amounts of sterile water from the nose to the outer edge of the eye.
543. You respond to the vehicle maintenance department, where a middle-aged man is flushing his eyes with tap water. He tells you that acid splashed in his eyes and that they are burning. Which of the following questions is the most important one to ask right away?
• Are you wearing contact lenses?
544. You are assisting another OEC technician to treat a young male with a deep laceration on his neck. Your partner appears to have controlled the bleeding. He asks you to apply an occlusive dressing. In addition to helping to control the bleeding, you recognize that another important benefit to this type of dressing on a neck wound is:
• Preventing the entry of air into the circulatory system.
545. A patient has sustained an injury to his mediastinum. Based on the anatomy of his chest, which of the following structures may have been injured?
• The lungs
546. The thorax is enclosed by the:
• Ribs, sternum, thoracic spine, and diaphragm.
547. The diaphragm separates the:
• Thoracic cavity from the abdominal cavity.
548. Which of the following actions occurs when the diaphragm and the intercostal muscles relax?
• Exhalation
549. The heart and lungs are located in the:
• Thoracic cavity.
550. Injuries at which level of the spinal cord can affect the phrenic nerves and therefore respiration?
• C3-C5
551. Which of the following statements indicates that the speaker understands chest trauma?
• Internal chest injuries are more difficult to assess and manage than are the more obvious external chest injuries.
552. A 39-year-old man has been stabbed once in the anterior chest. When notifying the patrol aid room by radio, you inform them that the patient has suffered what possible type of injury?
• An open chest injury
553. Which of the following phrases best describes a flail chest segment?
• Two or more adjacent ribs that have been broken in two or more places
554. Your patient was involved in a serious motor vehicle collision. Which of the following assessment findings best helps to determine that the patient has a flail chest segment?
• Paradoxical chest wall movement
555. You are managing a patient with a large flail segment in the right lateral chest. The immediate threat to this patients life is:
• Hypoxia.
556. An OEC candidate asks you to explain a pneumothorax. Your response should be that it occurs when:
• Air accumulates between the inner chest wall and the outside of the lung, causing the lung to collapse.
557. When assessing a patient, which of the following signs or symptoms is most indicative that the patient is suffering from a tension pneumothorax?
• Decreased breath sounds in the right lung
558. You have placed an occlusive dressing on a puncture wound on the right side of the chest of a 33-year-old woman. During your ongoing assessment, your primary concern is monitoring the injury for:
• Tension pneumothorax.
559. An OEC candidate indicates that she understands the difference between a pneumothorax and a tension pneumothorax when she makes which of the following statements?
• A tension pneumothorax can cause cardiac output to decrease; a pneumothorax does not.
560. You are treating a 26-year-old man with a pneumothorax. The patients breathing has been inadequate, and you have been assisting his ventilations with a bag-valve mask. The patients condition seems to be worsening. Because you understand the pathology involved in a pneumothorax, you are most concerned that:
• The pressure from the bag valve mask is causing more air to enter the pleural space and is causing a tension pneumothorax.
561. You suspect that a trauma patient is suffering from a hemothorax to the left lung. Which of the following assessment findings would reinforce your suspicion?
• Respiratory distress and the signs and symptoms of shock
562. Your 35-year-old patient has received major chest trauma. He complains of shortness of breath and pain. Your assessment reveals distended neck veins and pulsus paradoxus. Based on these signs and symptoms, you believe the patient to be suffering from:
• Pericardial tamponade.
563. You are instructing a class in Outdoor Emergency Care when a student asks you to explain the L.A.P. method of examining the thorax. Your best reply to this question would be which of the following statements?
• The LAP method directs you to look, auscultate, and palpate the chest.
564. You are following the LAP method to examine the chest of a 38-year-old trauma patient. To assess for thoracic cage instability, you would apply moderate downward pressure on the sternum while asking the patient to take a deep breath, and then you would:
• Apply moderate inward pressure on the lateral walls of the rib cage and ask the patient to take a deep breath.
565. You are listening to two patrollers discuss the accident to which they just responded. Which of the following injuries would lead you to believe that one of the patients had suffered from traumatic asphyxia?
• Bluish discoloration of the neck and face
566. You are at the scene of a shooting. Your assessment reveals a 23-year-old man who has been shot twice. The first wound is to the left lower quadrant of the abdomen and is bleeding. The second wound is to the left lateral chest and makes a sucking sound every time the patient takes a breath. The initial action of an OEC technician should be which of the following actions?
• Cover the chest wound with a gloved hand.
567. You are assisting at an accident scene and are asked to apply a dressing over a sucking chest wound on the patients left anterior chest wall. Which of the following items would you use?
• Vaseline gauze
568. You have applied a nonporous dressing to a puncture wound on a patients chest. An OEC candidate asks you why the dressing was taped only on three sides. Your best response is that taping the dressing on three sides:
• Allows trapped air to escape upon exhalation.
569. You treated a 19-year-old with a puncture wound to the chest by covering the wound with an impermeable dressing. Now, as you perform your ongoing assessment, you note that the patient is tachypneic and is complaining of difficulty breathing. Breath sounds on the side of the injury are also diminished. Which of the following actions should you take immediately?
• Lift a corner of the dressing from the wound for a few seconds; then reapply and check his breath sounds.
570. You are treating a 16-year-old male who was skiing out of control and ran off the trail and into the woods. When you arrive, he is alert and complaining of pain in his right lower chest. Upon examination, you note that he has impaled himself with a tree branch. He tells you he will feel better if you just remove the branch. Based on your training as an OEC technician, you would respond by making which of the following statements?
• We need to leave the branch in place until we get you to a hospital.
571. The abdomen is located between the:
• Diaphragm and the pelvic brim.
572. The function of organs in the abdomen is:
• Digestion, production of white blood cells, and the filtering of blood.
573. When palpating the anterior portion of a patients abdomen, you note tenderness in the left upper quadrant. You would recognize that which organ might be involved?
• The spleen
574. A patient complains of severe and sharp pain in the right shoulder when palpating the right upper abdominal quadrant. Based on this sign and location, what organ or structure is most likely involved?
• Liver
575. You have been called to aid a patient with generalized abdominal pain. The patient states that he recently had his gallbladder removed. You know that the gallbladder is located in the:
• Right upper abdominal quadrant.
576. Blunt trauma to the upper right abdominal quadrant can injure the:
• Liver.
577. Which of these organs is most fragile and, therefore, most likely to be injured by blunt trauma?
• Spleen
578. The spleen of a patient has ruptured. Because the spleen is a solid organ, you recognize that the primary threat to life is most likely:
• Blood loss.
579. The pancreas is best described as a:
• Solid organ.
580. The most common mechanism of injury to the pancreas is:
• Direct trauma to the middle of the abdomen above the navel.
581. You are presented with an 18-year-old male who had trauma to the middle of his abdomen during a collision. His difficulty breathing might be due to:
• Intestinal loops having moved into the thoracic cavity.
582. In which of the following injuries can the abdominal cavity migrate into the thoracic cavity?
• A tear in the diaphragm
583. An injury to the abdominal wall allowing the intestines to come out of the abdominal cavity is a(n):
• Evisceration.
584. You would recognize which one of the following situations as an acute abdominal emergency?
• A ski racer who hit a tree and has significant left flank pain prior to and during palpation
585. Which of the following statements indicates that you correctly understand the goal of assessing and managing a patient who has abdominal pain?
• It is more important to recognize a possible abdominal injury or emergency than it is to identify the exact cause.
586. When taking care of a patient with an abdominal injury, your first priority is:
• Keeping the scene, patient, and all rescuers safe.
587. A crying 42-year-old woman states that she has sharp abdominal pain that she localizes to her right lower quadrant. When assessing this patients abdomen, you would look for distention and symmetry and then:
• Start the exam by palpating the abdomen in the quadrant farthest away from the one that is painful.
588. When presented with a 6-year-old female with a stick impaled through her lower left abdominal quadrant, an OEC technician should:
• Stabilize the stick.
589. In most cases, the position of comfort in abdominopelvic trauma is:
• Supine, with padding under the knees to keep them slightly bent.
590. To prepare a patient with a pelvic fracture for transportation, you should:
• Apply a pelvic binder.
591. Because pelvic fractures are often associated with heavy internal bleeding, you should expect:
• To see hypovolemic shock.
592. You are assessing a conscious but confused hiker who became lost in the woods on a cold day. Your assessment shows that he has an open airway, adequate breathing, and a weak radial pulse. His skin is cold to the touch, and he is shivering. OEC technicians should recognize that the:
• Shivering is a protective means by which the body is attempting to warm itself.
593. A young man who was angry with friends wandered away from a party and spent the night outside uncovered in 40-50F. He is confused, has decreased but adequate breathing, and has a weak radial pulse. His skin is cool, and capillary refill is delayed. When looking to see if he is shivering, you note that he is not. Based on this presentation, OEC technicians can safely conclude that the:
• Patients body temperature is most likely dangerously low.
594. A young boy complains of pain in his fingers after spending several hours outside riding a sled in cold temperatures. After ensuring that he has no life-threatening conditions, you turn your attention to his hands and note that his fingers are cold, and the skin is pliable to the touch. Which of the following signs or symptoms would help confirm your suspicion that he is suffering from early or superficial frostbite?
• Soft and tingling skin on the fingers
595. Which of the following statements about immersion hypothermia is true?
• It can take more than 30 minutes for a person who has fallen into very cold water to become hypothermic.
596. The continued drop in core body temperature after removal from exposure to the cold is known as:
• Afterdrop.
597. The main cause of afterdrop is probably:
• The return of cold blood from the extremities.
598. Which of the following statements concerning immersion hypothermia is true?
• It can take more than 30 minutes for an individual to become hypothermic.
599. Which of the following instructions is most appropriate when moving a patient who has severe generalized hypothermia?
• "I want everyone to take extra care in moving her very gently to the stretcher; we do not want her to go into cardiac arrest."
600. When obtaining a history of a five-year-old boy with generalized hypothermia, which of the following questions provides the most important information?
• "Does he have any medical conditions?"
601. A patient with moderate hypothermia will have:
• A slow pulse and/or slow respirations.
602. You and your friends are camping on a 20F day. Which of the following assessment findings best illustrates that your bodies are no longer capable of compensating for the effects of the cold?
• Loss of fine motor coordination
603. Which of the following signs would you observe earliest in a hypothermic patient?
• Shivering
604. Which of the following findings best indicates that a patient with cold skin is suffering from moderate hypothermia?
• A heart rate of 52 beats per minute
605. Which of the following findings is a sign of severe hypothermia?
• Coma
606. You are with a candidate OEC technician who is attending to a snowboarder who has fallen. The candidate is very concerned that the snowboarder is shivering vigorously and asks you what this indicates. Which of the following statements would be your best response?
• Shivering is a sign of mild hypothermia.
607. A patient who is conscious and breathing has been pulled from a stream of cold water. To decrease her loss of heat via the mechanism of conduction, an OEC technician should immediately:
• Remove her wet clothing then cover her with blankets.
608. Prevention of heat loss in the care of a patient with a cold injury should include:
• Carefully removing wet clothing.
609. You have been called to aid an alert and oriented male patient whose friends state he spent several hours locked out of his house in cold temperatures. Given that the primary and secondary assessments have ruled out immediately life-threatening conditions, you have decided to rewarm the patient. Which of the following actions would be most appropriate for this patient?
• Apply hot packs to the patient's chest, groin, and armpits.
610. You need to begin rewarming an unresponsive patient who has a core temperature of 93F. Which of the following measures would most benefit this patient?
• Wrap the patient in several warm blankets.
611. When assessing the vital signs of a severely hypothermic patient, OEC technicians should:
• Check the patients pulse and respirations for up to one minute.
612. A severely hypothermic patient is in cardiac arrest. Most studies indicate that for patients who have been submerged in cold water for more than one hour, you should:
• Focus on rewarming and rapidly transporting the patient.
613. Which of the following statements indicates that an OEC technician understands the care of a patient with a localized cold injury to the foot?
• Warming a frozen body part can cause severe pain.
614. You are facilitating a talk on cold emergencies in the backcountry. You are asked when one should attempt to rewarm a hand or foot that is frostbitten. Which of the following statements would be your best response?
• Rewarming should take place only when there is no chance that the tissue will refreeze."
615. One of your friends on a hunting trip in Montana has suffered a severe localized cold injury to his hand and fingers. He does not have any life-threatening conditions, and you elect to rewarm the affected areas. Which of the following actions is most appropriate for your friend?
• Thaw and rewarm the tissue as quickly as possible.
616. After successfully rewarming a foot that has frozen toes, an OEC technician should:
• Cover the foot and toes with dry dressings.
617. A frostbitten skier stranded in the backcountry for three weeks may observe what happen to badly frostbitten toes?
• Autoamputation
618. It has taken you 45 minutes to extricate an avalanche victim. Based on your knowledge of hypothermia and avalanche victims, you know that:
• Hypothermia can occur after extrication due to a cold, windy environment.
619. The initial priority for avalanche victims is:
• Ensuring adequate ventilation.
620. The body cools itself primarily by:
• Evaporation.
621. You are at a football practice on a very hot summer day. A young player is panting and lying under a tree. His skin is flushed, and his teammates are continually wiping the sweat off his face, arms, and chest with towels and offering him fluids to drink. Which of the following statements is most appropriate for this situation?
• Its OK for him to sweat. It is helping his body cool down.
622. The most serious of the types of heat-related illnesses is:
• Heatstroke.
623. The signs of heat exhaustion are:
• Warm and very sweaty skin, elevated body temperature, headache.
624. You recognize that heat cramps are the probable cause of a patients problem when the patient makes which of the following statements?
• I have pain in my belly and legs.
625. The minimum body temperature of a patient for a diagnosis of heat exhaustion is:
• 101F.
626. Which of the following patients would have the highest priority for transport?
• A 19-year-old who is lethargic, has hot dry skin, and has tachycardia
627. You are at a park with your family when you hear a call for help. There is a teenager who is sick. On scene, you find a 16-year-old boy who is mildly confused. Friends state that they have been playing basketball for most of the morning and afternoon. The temperature is in the 90s, and the humidity is high. The boys airway is patent, and his breathing rate is elevated but adequate. His pulse is weak and rapid. Observation of his skin reveals it to be pale, cool, and moist. Which of the following would you do first for this patient?
• Lay him in the shade in a supine position, and elevate his feet.
628. You are treating a patient with suspected heat cramps. Which of the following treatments is the usual recommended treatment?
• Have the patient drink a mixture of 1/4 to 1/2 teaspoon of table salt in a quart of cool water.
629. When cooling a patient with a possible heatstroke, which of the following findings would be of most concern to a rescuer?
• Observing your partner giving the patient two aspirin to bring down his temperature
630. You are in the aid room with a patient who is unresponsive and has hot, dry skin. His friends state that he has been drinking and passed out in the hot sun for several hours. ALS has been called. Which of the following actions would be most beneficial to this patient now?
• Place cold packs on the patients groin and armpits.
631. You are teaching a basic OEC class and need to explain toxins. Which of the following statements is the best description of a toxin?
• It is a poison made by a living creature, including plants and animals.
632. A specific toxin or poisonous secretion of an animal that is usually transmitted by a bite or sting is known as:
• Venom.
633. Plants most commonly cause toxic reactions through which of the following routes?
• Ingestion and topical contact
634. On a camping trip with your family, your children ask if they can pick some plants for part of their wilderness dinner. Based on your OEC training, you explain to your children that they need to be careful picking wild plants and flowers, and that:
• Some plants can be very toxic, so it is best not to eat anything unless you are sure what it is.
635. Which one of the following groups is not included in the three main groups of mushrooms that are dangerous to humans?
• Big white mushrooms
636. Its early fall, and you have been backpacking on part of the Appalachian Trail with some friends. One member of the group comes up to you, holding some bright orange mushrooms that resemble pumpkins and asks if you think they are safe to eat. Based on your training, you would respond with which of the following statements?
• Those mushrooms can cause some pretty severe vomiting, diarrhea, cramps, and loss of coordination.
637. You will be taking a group of scouts on a weekend camping trip. You recognize that ticks are typically active at this time of year, so you want to teach the scouts about them. Which of the following statements about ticks is false?
• Tick bites are painful, so you will know when you have been bitten.
638. Which of the following signs and symptoms is not a characteristic of a bee, wasp, or hornet sting?
• Nausea and vomiting
639. Most reptile-related injuries are caused by:
• Snakes.
640. Marine life can cause varying degrees of injury and illness. Which of the following marine creatures can cause respiratory failure and cardiovascular collapse in humans?
• Jellyfish
641. Attacks by which of the following creatures do not cause both soft-tissue injuries and fractures?
• Swordfish
642. You are working at the first-aid station at the local scout camp. A counselor brings in two young boys that he thinks were playing in an area where poison oak was found. Which of the following actions would not be a part of your treatment for the possible exposure to poison oak?
• Applying a topical cream such as bacitracin to the boys hands and arms
643. Your neighbor has been cleaning up the wooded area around his yard and burning brush and weeds. He tells you that he is trying to rid the area of poison ivy. You recognize that his actions could:
• Cause significant respiratory problems for anyone exposed to the smoke.
644. When a young child at a neighborhood cookout cries out that he was stung by a bee on his forearm, you should:
• Scrape the stinger off the skin with a firm, flat object such as a credit card and then apply ice to reduce the swelling and pain.
645. While hiking in the Rockies, your friend is bitten on the arm by a rattlesnake. You should quickly:
• Immobilize the arm and place it lower than your friend's heart.
646. Which of the following actions is not an appropriate treatment measure for a sting by a marine creature?
• Rubbing the affected area to remove any spines
647. When caring for someone who has been attacked by a large animal such as a moose or bear, you should assume that:
• The potential for spinal injury exists.
648. The height or vertical elevation above a fixed point is known as:
• Altitude.
649. The concentration of oxygen at sea level is approximately:
• 21%.
650. As altitude increases:
• The proportion of oxygen decreases.
651. You are teaching a class on acute mountain sickness. When one of your students asks you to explain the term acclimatization in relation to AMS, your best response would be which of the following statements?
• Physiologic adjustments that increase the delivery of oxygen to cells.
652. Which of the following processes does not occur during acclimatization?
• Dilation of pulmonary blood vessels
653. You are working at a mountain resort and are presented with a 50-year-old woman who states that she does not feel well. From your assessment, which of the following findings would seemingly indicate that the patient is being adversely affected by the high altitude?
• A complaint of feeling short of breath on exertion
654. In an otherwise healthy individual, the presence of a headache and feelings of sickness at high altitude is known as:
• Acute mountain sickness.
655. Labored breathing at rest and audible chest congestion herald the development of a serious, potentially life-threatening stage of what altitude-related condition?
• HAPE
656. You are with a group of hikers on the third day of an 11,000-foot mountain ascent. One of the hikers has not been feeling well for a couple of days. Today, members of the group notice that he is having difficulty getting dressed and speaking. From your training in outdoor emergency care, you recognize that these signs and symptoms are most often associated with:
• HACE.
657. The most effective method for preventing high-altitude illness is to:
• Make gradual ascents.
658. You are asked to speak to a college group that is planning a ski trip to the Alps. In order to help them plan for reducing their risks for developing altitude-related illnesses, you suggest all of the following except:
• Do a lot of heavy physical exertion early in the trip to help them acclimate.
659. Your family is planning a ski trip to a resort that is at about 10,000 feet of elevation. You suggest that you spend the first two nights of the trip at a hotel where the elevation is about 6,500 feet. When your children protest that they are in good shape and want to go immediately to the resort, you explain that you are concerned about altitude sickness and:
• Physical fitness does not necessarily prevent altitude sickness.
660. You are working at the summit of a 9,000-foot mountain. A 60-year-old woman is brought to you, complaining of headache, fatigue, and shortness of breath. You prepare to do your assessment and recognize that your goal is to:
• Determine whether this is an emergent condition so that you can initiate life-saving treatment.
661. Given that 50 percent of patients with HAPE also have symptoms of AMS, it is important that you ask patients if they have had which of the following groups of signs/symptoms?
• Fatigue, nausea, difficulty sleeping
662. In order to assess for a key symptom of HACE, you would ask patients to:
• Walk a straight line heel to toe.
663. A patient at a mountain ski resort has notable shortness of breath. She denies any past medical history and takes no medications. After applying oxygen, you realize that the fundamental treatment to helping this patient improve is to:
• Descend to a lower altitude.
664. You have a 32-year-old male patient who just arrived at the aid room on the top of a 9,000-foot peak. He is exhibiting signs of altitude sickness and HAPE. You recognize that the final key to a successful outcome of treatment is to:
• Recognize the patients signs and descend to a lower elevation.
665. Your group of climbers has reached an elevation of 8,500 feet. One of the climbers is increasingly short of breath and now has audible chest congestion. Your party has a limited supply of oxygen, which you immediately apply to the climber at 15 LPM via a nonrebreather mask. Based on your assessment that the climber has HAPE, you recognize that the next necessary treatment is:
• A rapid descent of at least 1,500 to 3,000 feet.
666. When treating an AMS patient, use high-flow oxygen to keep saturation at least what percentage?
• 94%
667. You are accompanying a team of hikers up a high mountain. The next morning, you are summoned to a tent and find one of the climbers confused and complaining of a headache. His airway is patent, and his respirations are 24 per minute. He has no medical history and was in good health until found ill this morning by his friend. Suspicious of high-altitude cerebral edema (HACE), which of the following actions should you take?
• Provide high-flow oxygen
668. The worldwide number of drowning deaths reported each year is in the range:
• 300,000-400,000.
669. The final event in the series of events in drowning is:
• Cardiac arrest.
670. The mammalian diving reflex is most prominent in:
• Young children.
671. Which of these is a submersion injury?
• Someone strikes a submerged rock while swimming.
672. Which of the following strategies does not limit or mitigate risk in water-based activities?
• Entering swiftly moving water accompanied by a buddy
673. Drowning is defined as:
• Suffocation by submersion in a liquid.
674. The first thing to happen when a person drowns is usually:
• Panic.
675. Trauma from a dive injury that results in tissue damage within any air-filled structure of the body is called:
• Barotrauma.
676. Decompression sickness or the bends is a(n):
• Buildup of nitrogen bubbles in the body
677. When arterial gas embolism (AGE) occurs, the gas within the lungs:
• Expands, rupturing alveoli.
678. The cause of an arterial gas embolism (AGE) is:
• A rapid ascent by a diver.
679. Decompression sickness can cause:
• Severe muscle and joint pain.
680. A large nitrogen bubble can act as a(n):
• Embolus that blocks blood flow.
681. Squeeze is a term for excessive external pressure on various parts of the body. Reverse squeeze:
• Is too much pressure from within a body compartment or organ.
682. Assessment of patients with water-related emergencies is not remarkably different from assessment of patients with other emergencies. OEC technicians may:
• Need to don a personal floatation device.
683. During assessment of the water-related emergency for any submersion injury, care should be taken to:
• Protect the patients spine to prevent additional neurological injury.
684. For any submersion injury that is not a life-threatening water-related emergency, a complete secondary assessment is performed using:
• DCAP-BTLS.
685. Patients with arterial gas embolism or decompression sickness should be transported by ground to a facility that:
• Has a hyperbaric chamber.
686. A patient who appears dead due to a deep cold-water drowning should be:
• Transported to a medical facility while being given CPR.
687. A patient who is conscious and breathing has been pulled from a cold stream. To decrease the patients loss of heat via the mechanism of conduction, OEC technicians should immediately:
• Thoroughly dry the patient and then apply a blanket.
688. A 9-month-old boy has been stung on his tongue after sucking on a bottle that had a bee on the nipple. In comparison to the same injury in an adult, why would an OEC technician be more concerned over this child?
• A childs tongue is proportionally larger, increasing the chances of airway occlusion from even minor swelling.
689. Fontanels, which are openings in the skulls of newborns that allow brain expansion, close when the child is at approximately what age?
• 18-20 months of age
690. An OEC technician asks you why you should not overextend the airway when performing a head tilt-chin lift maneuver on a pediatric patient. Which of the following replies would you make?
• The cartilage of the trachea is very soft and can compress if the neck is extended too far.
691. You are treating a 2-year-old who fell down a flight of stairs. You are concerned about a head injury and know that toddlers have a higher risk for traumatic brain injury than adults because:
• A toddlers fontanelles will not close until the child reaches preschool age.
692. Children exhibit great bursts of energy followed by sudden profound fatigue because:
• They have fewer energy reserves than adults.
693. You are assessing a 2-week-old baby who is sick. Assessment reveals that he has a fever and difficulty breathing. Which of the following additional assessment findings would be most concerning to you given the age of this patient?
• Nasal passages that are occluded by mucus
694. An adolescent is defined as someone who is:
• 13-18 years old.
695. You have been called to care for a 21-month-old girl who has been bitten by a dog. Given the patients age, you would appropriately classify the patient as:
• A toddler.
696. You are reviewing psychosocial development of a preschooler in your OEC class. You recognize that preschooler includes children who are:
• 3-5 years old.
697. A mother brings her 3-year-old son into the aid room. She says he has a low-grade fever and seems to be having slight difficulty breathing. Which of the statements by the mother would make you think the child may have croup?
• Last night, he had a barking-like cough.
698. A 4-year-old girl is sitting upright on her mothers lap with her chin thrust forward. She looks lethargic and is drooling. Her airway is open, and she appears to be breathing adequately. Her mother says she has had a fever and is complaining of a sore throat. You are very concerned that she may be developing:
• Epiglottitis.
699. Cardiac arrest in children is most commonly caused by:
• Respiratory failure.
700. You have been called to assist a panicked mother who is worried about her son. When you arrive, she tells you that her 4-year-old son has been quiet all morning and napping while the rest of her children were skiing. While napping he began to shake all over for about 30 seconds. The child is now resting quietly and has adequate respirations. His radial pulse is strong, and his skin is very hot and moist to the touch. Based on this presentation and information, you assume that the seizure occurred secondary to:
• A fever.
701. Which of the following conditions is not a possible cause of seizures in young children?
• Hypothermia
702. You read in the newspaper about a 9-month-old child who died from sudden infant death syndrome (SIDS). From your training, you know that this means that the infant:
• Died unexpectedly and of an undetermined cause.
703. The increased pliability of the ribs of children makes them more prone to:
• Bruising of the lung.
704. Which of the following statements about shaken baby syndrome is false?
• It rarely occurs in upper-class families.
705. Bilateral injuries, circumferential bruising, and pattern bruises are suggestive of:
• Child abuse.
706. When assessing a 3-year-old child for possible injuries after the mothers boyfriend said the child fell down a flight of stairs, which of the following findings would raise your suspicion that the child may be a victim of physical abuse?
• Circumferential bruising is apparent on the childs arm.
707. Which of the following statements is most appropriate concerning dealing with caregivers and children during a medical emergency?
• I try to include caregivers in all that I do with their child so that the child and the caregivers are more comfortable.
708. The American Academy of Pediatrics recommends the use of the Pediatric Triangle to quickly determine if a child is sick or not sick. This method allows rescuers to quickly assess all of the following about the patient except his/her:
• History of illness or injury.
709. Which of the following behaviors would an OEC technician recognize as uncharacteristic of a conscious and stable 2-year-old boy who fell and hurt his hand?
• He does not cry or protest when you take him from his mother to assess him.
710. While following the Pediatric Triangle of assessment, you observe an infant and note that she is paradoxically irritable. This sign is often indicative that the child is:
• Very ill.
711. When seen in a child, the tripod and sniffing positions are usually signs of:
• Respiratory distress.
712. You are assessing an infant who has been ill and has had a fever for the past two days. As you approach the infant, you note that she has grunting respirations. Based on your OEC training, you determine that grunting is:
• A symptom of severe respiratory disease.
713. You must assess the pupils of a 5-year-old boy who fell. Which of the following statements would be most appropriate for you to make before performing the assessment?
• I am going to use this light to look into your eyes.
714. You are assessing a 2 and half year-old child who was involved in a minor car collision. She is currently alert and oriented. While you are assessing her for possible injuries, which of the following actions would be considered most appropriate?
• Allow the child to hold a favorite toy during the assessment.
715. You have been called to care for an unattended 6-year-old girl who has vomited once and is complaining of mild abdominal pain. When you are performing the physical assessment and obtaining a SAMPLE history, which of the following approaches would be considered most appropriate?
• Allowing her to play with your stethoscope before listening to her lungs
716. When assessing a 3-year-old child with a respiratory illness, which of the following assessment findings would be least concerning to you?
• A respiratory rate of 28 breaths per minute
717. You are night skiing when you are notified that a child has been involved in a collision on the intermediate trail. As you approach the scene, you are thinking about your assessment and realize that the cold and low light may make assessing the childs circulation difficult. In this situation, which of the following is least likely to be effective?
• Fingertip capillary refill
718. Which of the following instructions would you give an OEC technician who is preparing to assess a stable 9-month-old boy with a rash?
• Have the mother hold him as you do the assessment.
719. You are treating a child who has a minor head laceration that is bleeding profusely. Which of the following statements would show your partner that you understand bleeding in children?
• Children have a smaller blood volume than adults, so this rate of bleeding is serious.
720. Which of the following statements regarding the treatment of a pediatric patient in a prehospital setting is true?
• If a child looks sick and is not getting better with care, assume that the child is getting worse.
721. You are immobilizing a 4-year-old boy on a long spine board. Which of the following actions would be appropriate when performing this intervention?
• Place padding between the patients shoulders and the spine board.
722. An infant who is short of breath is alert and has adequate respirations at a rate of 54 per minute. His skin color is pink but slightly cool to the touch. When you place a pediatric mask on his face, he becomes very upset and begins to physically struggle to remove it. In this situation you would:
• Allow the mother to hold the infant and then provide blow-by oxygen therapy.
723. You are called to assist a toddler who has just had a seizure that lasted about 60 seconds. His mom reports that the child has a history of seizures. Your assessment reveals that he is now responding to painful stimuli, is breathing normally, and has a normal radial pulse. Which of the following actions would you take?
• Place him in a recovery position and continue to assess him.
724. You have been asked to deliver a talk about the geriatric population to the new OEC class. Which of the following points would you emphasize in your presentation?
• Many elderly individuals have a combination of different diseases in various stages.
725. A new OEC technician asks you why the elderly are at higher risk for developing pneumonia than younger individuals. You inform him that the elderly are more susceptible to respiratory infections because of:
• A reduced cough reflex.
726. The son of an 88-year-old patient states that he told his mother to increase the amount of an antibiotic she was taking so she would "feel better faster. This represents a problem because the elderly have:
• Decreased liver and kidney function.
727. By age 85, the brain of a typical geriatric patient can shrink by as much as 10 percent due to:
• A decrease in brain cell numbers.
728. In a typical geriatric patient, a reduction in cerebral blood flow can:
• Reduce the amount of glucose and oxygen that reaches the brain.
729. Which of the following statements shows that an OEC technician has an accurate understanding of vital signs in relation to geriatric patients?
• Respiratory capacity is greatly reduced in geriatric patients."
730. For many geriatric patients, food is less appealing because:
• Their sense of smell is reduced, and they have fewer taste buds.
731. The lung capacity in a healthy 90-year-old patient is ________ that of a 30-year-old patient.
• Half of
732. Some elderly patients have osteoporosis, a condition that results in:
• Decreased bone density.
733. For the same individual, on average, blood pressure at age 30 compared to blood pressure to age 70 will be:
• Lower.
734. In geriatric patients, strokes can be caused by a blockage or by:
• A rupture of a cerebral blood vessel.
735. Family members tell you that three hours ago, their 76-year-old mother suddenly became confused and had great difficulty speaking. However, within 15 minutes, she returned to normal. Based on this description, an OEC technician should be suspicious of:
• A transient ischemic attack.
736. Which of these is a common cause of COPD?
• Chronic bronchitis
737. Over 60% of abdominal pain problems in geriatric patients require:
• Surgical intervention.
738. Many elderly patients participate in a practice called polypharmacy, which is the:
• Simultaneous taking of multiple medications.
739. To identify all the drugs, prescriptions, herbal supplements, and over-the-counter drugs that a patient may be taking in combination, OEC technicians should use:
• SAMPLE.
740. The class of medications that is most commonly prescribed to elderly patients is:
• Cardiovascular medications.
741. One group of drugs prescribed for geriatric patients is beta-blockers. This type of drug:
• Manages cardiac arrhythmias.
742. Geriatric patients may also have a prescription for a diuretic such as HCTZ or Lasix. The role of this type of drug is to:
• Decrease the volume of fluid circulating in the cardiovascular system.
743. The vital signs of elderly patients with excessive internal or external bleeding may not provide an indication of shock:
• If these patients are taking beta-blockers or calcium channel blockers.
744. Compared to younger patients, trauma in geriatric patients results in:
• A higher mortality rate.
745. Correcting a life-threatening condition such as external bleeding in elderly patients can be more problematic because:
• Many elder patients are taking warfarin or other blood thinners.
746. Which of the following statements about traumatic injuries in the elderly is true?
• Injuries are most commonly caused by falls.
747. Falls result in ____% of all deaths in the geriatric population.
• 12
748. Which of the following conditions must OEC technicians consider when evaluating hypotensive geriatric patients with altered mental status who show no signs of external bleeding, chest injury, or abdominal trauma?
• A hip or pelvic fracture
749. An elderly woman has fallen down three steps and is complaining of back pain. After you perform a primary assessment, which of the following questions should you ask to elicit the next important piece of information?
• "Did you become dizzy before you fell?"
750. Compared to younger individuals, geriatric patients who fall have a higher incidence of fractures to the _____ region of the spine.
• C1-C2
751. Which of the following statements concerning insulin pumps is false?
• They are implanted in the patients chest.
752. An advance directive is a:
• Legal document that provides medical direction for life-saving efforts.
753. A hearing-impaired patient wearing a hearing aid is having a great deal of difficulty hearing your questions. Which of the following actions should you take first?
• Ensure that their hearing aid is turned on.
754. When communicating with a geriatric patient, OEC technicians should:
• Use laymans terms.
755. Which of the following strategies is best for OEC technicians for communicating with a geriatric patient who has diminished eyesight?
• Stand in front of the patient and talk calmly.
756. An effective way to interview a geriatric patient is to:
• Use good listening skills.
757. An elderly trauma patient may struggle with mental clarity, so OEC technicians should try not to:
• Ask open-ended questions.
758. You and your partner respond to the lodge to aid an elderly woman who is not feeling well. When you perform a secondary assessment:
• Only one of you should ask the patient questions to avoid confusion.
759. You have been called to an unknown medical emergency. On scene, you are presented with an 84-year-old patient who is confused and does not obey commands. Which of the following action would be most useful in determining the patient's normal mental status?
• Question the patients family members.
760. When taking the pulse of an elderly patient, it is recommended that OEC technicians:
• Take the pulse on both sides and compare the two rates.
761. You have been called by the caregiver of a 91-year-old woman with dementia. The patient is complaining of a possible fracture to her upper left arm. As you talk to the caregiver and note various bruises on the patients body, you become suspicious of elder abuse. Which of the following actions is most appropriate at this time?
• Treat the patient for a possible broken arm.
762. You are assisting a 29-year-old spinal injury paraplegic skier who lost control of his sit ski and hit a tree. Even though there are no obvious signs of injury, the patient suddenly starts to feel panicky, and you become concerned about autonomic dysreflexia. Other signs/symptoms that would support this assessment include all of the following except:
• Hypotension.
763. A group of ski patrollers have invited you to join their team on a bike ride to raise money for multiple sclerosis (MS) research. You recall learning about MS in your OEC class, but you ask your friend to remind you of some of the key features of this condition. Which of the following statements made by your friend would be most accurate?
• MS is a progressive disease that causes degeneration of both central and peripheral nerves.
764. What is true about a patient with a cognitive disability?
• All of these are correct.
765. When caring for a patient with dyslexia, it is important to remember that:
• The patient may have difficulty processing new information.
766. You are teaching a class on adaptive athletes and are asked to discuss Asperger syndrome. Which of the following statements is an appropriate response?
• Asperger syndrome is an autism spectrum disorder in which the person may be fearful in unfamiliar environments.
767. You are caring for a hearing-impaired athlete and realize that in order to communicate, you:
• Should face the person because most hearing-impaired individuals can read lips.
768. Its 2 p.m. on Saturday, and youre out for a bike ride with friends. About halfway down a small hill, you see an adult and a child at the side of the road. It appears that the child may have fallen. As you approach, you ask if you can help, to which you hear a moms thankful yes. The child, who is about 8 years old, is sitting quietly and does not respond when you say hi. The mother explains that her son has atypical autism. Based on this information, which of the following techniques might you use to complete your assessment?
• Give the child simple, one-step directions.
769. When caring for patients with life-long intellectual disabilities:
• Include their caretakers in the assessment because they can help with communication.
770. You are assisting a 32-year-old woman whose right leg has been amputated below the knee. The injured skier is complaining of severe pain in her right thigh, which shows evidence of a fracture. After picking up a rigid splint, what should be your next thought?
• I should use a quick splint instead.
771. It is a cold, snowy Saturday afternoon, and, unfortunately, the chair lift has broken, and a lift evacuation is required. You know that there is an adaptive skier group somewhere on the lift. As you prioritize the lift evacuation, you recognize that:
• Athletes with mobility problems are at increased risk of cold-exposure injuries.
772. Your OEC class asks you to describe the term behavior. Which of the following statements would be the correct response?
• What is acceptable behavior in one community may not be acceptable in another.
773. Your first priority in managing a patient with a behavioral emergency is:
• Ensuring your own personal safety.
774. Which of the following statements about behavioral emergencies is true?
• The person acts in a way that is not tolerable to those around them.
775. Which of the following patients should an OEC technician consider a potential behavioral emergency?
• A 29-year-old man with diabetes who is combative and refusing care
776. Two of the most common medical causes of abnormal behavior are:
• Hypoxia and hypoglycemia.
777. Which of the following patients is exhibiting signs of psychosis?
• A 37-year-old who is having hallucinations
778. A condition in which a person exhibits abnormal behavior but is able to function within the normal boundaries of reality is known as:
• A mood adjustment or affective disorder.
779. You have stopped a male skier and a female skier who were racing down a closed trail. Initially, the man was calm, but suddenly he becomes angered and says that you cant tell me what to do. When you try to talk with him, he pulls out a knife and tells you to get away or Ill kill you. The female skier is frightened and states that her friend sometimes gets very aggressive when he drinks. You put in a call for security and additional staff, but you know it will be a few minutes before they arrive. Which of the following statements describes your best course of action in the interim?
• Remain a safe distance from the man and let him know you are there to help.
780. You are describing techniques for remaining safe when responding to a call for a patient with a behavioral emergency. Which of the following statements is appropriate?
• "You should be sure that you have at least one escape route, and do not let the patient get between you and that way out.
781. Which of the following statements shows that an OEC technician has a good understanding of assessing a patient who is having a behavioral emergency?
• Try to assess patients from a safe distance until they start to answer your questions calmly.
782. Which of the following behaviors is a sign of potential aggression in a patient?
• The clenching of both fists
783. Which of the following pieces of information is most important to an OEC technician in assessing a patient with abnormal behavior?
• Whether or not the patient is dangerous
784. Which of the following actions is best to keep a behavioral patient calm and to avoid provoking a violent outburst?
• Inform the patient of everything that is happening and use a calm voice throughout care.
785. You and a fellow OEC technician are discussing how to talk with someone experiencing a behavioral emergency. Which of the following statements indicates that you have a good understanding of the correct approach?
• Demonstrate active listening skills by repeating or paraphrasing what the patient said.
786. You are called to the lodge to assist a 25-year-old man who reportedly is having hallucinations. When you arrive, he is sitting quietly and appears calm. Information obtained in your SAMPLE history indicates that the patient started having hallucinations yesterday. He was diagnosed with schizophrenia a few years ago and has been prescribed two different medications. Which of the following questions is most pertinent to the situation and should be asked next?
• Have you been taking your medications as prescribed?
787. Which of the following situations is an indication for the use of restraints?
• The patient is in imminent danger of harming himself.
788. When treating a patient exhibiting abnormal behavior, you should:
• Assume until proven otherwise that the source of the abnormal behavior is an underlying medical condition.
789. Which of the following statements regarding the use of physical restraint is true?
• An OEC technician must follow local protocols for restraining patients.
790. You are out with a group of friends in a local tavern. One of the patrons gets argumentative and will not calm down. By the time the police arrive, the patron has gotten combative and is breaking glass bottles. The police subdue the patron and have him restrained. As you watch all of this happening, which of the following situations would cause you to be concerned?
• The police have restrained the patron in a prone position on the floor.
791. When mechanically restraining a patient, you should:
• Use a method consistent with your local protocols.
792. Special operations is a term used to denote:
• Infrequently performed activities that require specialized training, skills, and equipment in a remote setting.
793. You overhear a group of EMTs discussing the concept of driving an ambulance with due regard for the safety of others. Which of the following statements indicates that the EMT understands the concept?
• When approaching an intersection, I wait for all traffic to stop before proceeding through.
794. Which of the following statements regarding an ambulance parked at an incident is true?
• The area around the ambulance and the accident scene is known as the ambulance operation zone.
795. Which of the following tasks is not a task an OEC technician should be ready to perform in assisting an EMT in the back of an ambulance?
• Medication administration
796. You and your friend arrive at an accident scene where a compact car has rolled down an embankment. Based on your OEC training, you realize that before anyone enters the vehicle, it should be stabilized. Vehicle stabilization includes all of the following actions except:
• Having several men hold the vehicle to prevent it from moving.
797. Which organization helps create Community Emergency Response Teams?
• The Incident Command System
798. Organized under the DHHS, community-based volunteer medical professionals who have agreed to help in the event of a disaster are known as the:
• Medical Reserve Corps.
799. HAZWOPER training is required by the federal government for anyone who works in an environment in which uncontrolled hazardous materials may be encountered. Which is the correct word for a letter in HAZWOPER?
• O: operations
800. Which of the following statements regarding the NFPA safety diamond is true?
• The NFPA safety diamond identifies the known safety hazards of a given material.
801. Which of the following statements concerning HAZWOPER training is false?
• All EMS personnel are trained in all aspects of a HazMat-related incident.
802. In a hazmat incident, a boundary that is set up to prevent members of the public from entering the incident is called a(n):
• Isolation perimeter.
803. The contaminated area in a hazardous incident is known as the:
• Hot zone.
804. In a HazMat incident, the transition area in which decontamination occurs is known as the:
• Warm zone.
805. You are instructing an OEC class on the handling of a hazmat incident. You are asked where OEC technicians would be assigned if they were assisting at a hazmat scene. Which of the following statements is the appropriate response?
• Because of your OEC training, you would probably be assigned to the cold zone.
806. Toxic nerve agents:
• Overstimulate the nerve endings and the central nervous system.
807. In the event that OEC technicians are exposed to an organophosphate or a chemical nerve agent, they may need to self-administer the nerve agent antidote. Which of the following statements regarding the self-administration of a nerve agent antidote is false?
• Depending on the symptoms observed, the initial treatment will either be an atropine auto-injector or a 2-PAM Cl injector.
808. Which of the following activities is not considered a special operation?
• Toboggan transportation
809. Search and rescue operations consist of which of the following five basic tasks?
• Mobilization, intelligence gathering, containment, search, and rescue
810. Logistics, one of the four basic functions of an organized avalanche rescue, is primarily concerned with:
• Caring for the rescuers in the field.
811. Which of the following lists correctly places the tasks of fire ground operations in order of priority?
• Size-up, accountability, rescue/life safety, confinement, control, ventilation, property conservation